Skip to main content

All MCQs CA Course - May 2019 Paper with Changed pattern

Hello everyone, following are various Multiple choice questions from different subjects. I Hope you will like it

Happy reading

Section Z - ISCA MCQ and Important One Words

Paper 6: Information Systems Control and Audit (Old Course) Official ICAI MCQ

1. Arrange in chronological order of their assessment.
a. Risk
b. Threat
c. Vulnerability
d. Impact

(a) a,b,c,d
(b) c,b,a,d
(c) d,c,b,a
(d) c,b,d,a

2. Complete the sentence. "_______ " is not a RISK management strategy.
(a) Define
(b) Eliminate
(c) Share
(d) Mitigate

3. COBIT 5 principles include all except,
(a) Meeting Stakeholder Needs
(b) Covering Enterprise End To End
(c) Separating Governance From Management
(d) Enabling Better Controls

4. Creating a Governance, Risk and Compliance (GRC) framework is responsibility of ____________.
(a) Management
(b) Auditors
(c) Board of Directors (BoD)
(d) Auditor and BoD

5. Best definition to define a HUMAN being in terms of System.
(a) Physical, Probabilistic, Manual, Close
(b) Physical, Probabilistic, Manual, Close / Open
(c) Physical, Probabilistic, Automatic, Close
(d) Physical, Probabilistic, Manual, Open

6. Moving the Information System triangle from TOP to BOTTOM, kindly arrange in sequence which system
user shall encounter first to last.
(a) MIS, DSS, ESS
(b) TPS, KMS, DSS, EIS
(c) EIS, DSS, KMS, TPS
(d) TPS, KMS, DSS, MIS

7. A Company ABC Ltd. is facing trouble paying incentive to its sales representatives. The incentive is paid on
the basis of sales turnover achieved region-wise. Company has appointed you as a consultant asking your
input as to which type of system they need to create to solve the problem. Please guide.
(a) MIS
(b) EIS
(c) TPS
(d) ESS

8. Sales persons at BIG BAZAR use bar code scanners at the time of billing. This system of reading bar codes
for billing can be best classified as _______.
(a) TPS
(b) OAS
(c) MIS
(d) ESS

9. An accountant changes the voucher dates in TALLY without proper authority and sanction by management.
This act by accountant shall be best classified as breach of which key principle?
(a) Integrity
(b) Confidentiality
(c) Availability
(d) Confidentiality, Availability and Integrity.

10. In an accounting system, an accountant has rights to create as well as modify vouchers. This feature raises
the risk assessment by auditor to higher level. As an auditor, you need to define which principle has not
been followed?
(a) Confidentiality
(b) Segregation of Duty
(c) Availability
(d) Integrity

11. TALLY accounting software gives a warning when cash balance may turn negative on updating a voucher.
This is good example of ______.
(a) Compensatory Control
(b) Detective Control
(c) Corrective Control
(d) Preventive Control

12. Security guard at an ATM is an example of ______.
(a) Compensatory Control, Logical Control
(b) Detective Control, Physical Control
(c) Preventive Control, Physical
(d) Corrective Control, Logical Control

13. To use a GAS based fire suppression system, what needs to be done first.
(a) Call Police
(b) Call Medics
(c) Evacuate Humans
(d) Remove Computers

14. Business Continuity Planning by nature is a ______.
(a) Compensatory Control
(b) Detective Control
(c) Corrective Control
(d) Preventive Control

15. A large company is in process of creating a BCP framework. The Board of Directors have appointed you as
a BCP consultant. Board has asked you to justify / reason out the main objective of BCP is to prevent /
minimize losses. Your report shall highlight the reason for having BCP except this reason..
(a) Revenue Loss
(b) Reputation Loss
(c) Productivity Loss
(d) New Customer Acquisition
16. Arrange the following BCM process chronologically. A. BCM Strategies B. Information Collection
C. BCM Training D. Testing and Maintenance E. Development and Implementation
(a) A,B,C,D,E
(b) B, A, E,D,C
(c) B, A, E,C,D
(d) E,D,C,B,A
17. BCP includes various plans to be prepared. These include Emergency Plan, Back-up Plan, Recovery Plan
and Test Plan. BCP as a process has to start before disaster, as an entity without a BCP shall not be going
concern after disaster. All plans are drafted, documented before disaster. Of the four plans, which are those
where major part of plan is executed before a disaster occurs.
(a) All Four
(b) Emergency Plan, Back-up Plan
(c) Back-up Plan, Test Plan
(d) Test Plan, Recovery Plan
18. Which of the following back-up is most costly to have?
(a) Full
(b) Incremental
(c) Differential
(d) Both Incremental, Differential
19. DRDO (Defense Research and Development Organization, GOI) developed India's own fighter jets by name
TEJAS. TEJAS, is considered to be one of the most light and least costly fighter jet in its class. Which
system development method was used by DRDO. (HINT: The fighter jets were being developed for the first
time in India).
(a) SDLC
(b) Prototype
(c) Incremental Model
(d) Spiral Model
20. Which system development model is considered to be most complex?
(a) Waterfall
(b) Prototype
(c) Incremental Model
(d) Spiral Model
21. Which of the following is not an audit risk?
(a) Inherent Risk
(b) Country Risk
(c) Detection Risk
(d) Control Risk
22. Auditor, to check whether the control in an accounting application restricting payment in excess of
Rs.10,000/- in cash is working or not, creates a dummy voucher. This act of auditor would classify as which
concurrent audit technique?
(a) SNAPSHOT
(b) ITF
(c) SCARF
(d) CIS
23. Section 2(zh) of the Information Technology Act, 2000 (As amended 2008) defines "Verify". The definition
establishes two important aspects for electronic records, namely ___________ and ___________.
(a) Non-Repudiation, Integrity
(b) Confidentiality, Integrity
(c) Availability, Integrity
(d) Confidentiality, Non-Repudiation
24. Company has outsourced its accounts department. Statutory Auditor of the company shall be taking help
from which auditing standard to complete the audit?
(a) SA 300
(b) SA 402
(c) SA 500
(d) SA 510
25. Which of the following is not an example of Cloud computing model?
(a) IaaS
(b) PaaS
(c) AaaS
(d) SaaS

ISCA Q4 Google App Engine is an example of ___________.
(a) Network as a Service (NaaS)
(b) Platform as a Service (PaaS)
(c) Infrastructure as a Service (IaaS)
(d) Communication as a Service (CaaS)

(b) Platform as a Service (PaaS)

ISCA Q7. Which of the following step is not involved in an Application Access Control mechanismprocess?
(a) Identification
(b) Authentication
(c) Confidentiality
(d) Authorization

(c) Confidentiality

ISCA Q8 What does ACID Test in a Transaction Processing System (TPS) stand for?
(a) Atomicity, Confidentiality, Isolation, Durability
(b) Authenticity, Confidentiality, Integrity, Durability
(c) Authenticity, Consistency, Integrity, Durability
(d) Atomicity, Consistency, Isolation, Durability

(d) Atomicity, Consistency, Isolation, Durability

ISCA Q9 ___________ is defined as an inherent weakness in the design, configuration, or implementation of a network or system that renders it susceptible to a threat.
(a) Threat
(b) Vulnerability
(c) Risk
(d) Exploit

(b) Vulnerability

ISCA Q11 Which of the threat is not covered under Security Management Controls?
(a) Natural Calamities
(b) Structural Damage
(c) Pollution
(d) Hackers

(a) Natural Calamities

ISCA Q18 Arrange the following phases of System Development Life Cycle (SDLC) in order. (i) Investigate (ii) Maintenance and Review (iii) Analyse (iv) Implement (v) Design
(a) (i), (ii), (iii), (v), (iv)
(b) (i), (iii), (v), (iv), (ii)
(c) (ii), (i), (iii), (v), (iv)
(d) (i), (iii), (iv), (v), (ii)

(b) (i), (iii), (v), (iv), (ii)

ISCA Q19 Audit Trail is an example of ____________ Control.
(a) Detective
(b) Application
(c) Preventive
(d) Corrective

(a) Detective

ISCA Q20 Which of the following does not constitute part of Phase II: System Analysis of System Development Life Cycle (SDLC)?
(a) Observation of current procedures
(b) Sending out questionnaires
(c) Interviewing staff
(d) Technical Feasibility

(d) Technical Feasibility

ISCA Q22. In a proposed system, entity finds that system does not have an option to modify rates of TDS. The need may arise whenever law changes in future. This failure may be defined as failure of…
(a) Economic Feasibility
(b) Legal Feasibility
(c) Operational Feasibility
(d) Schedule Feasibility

(b) Legal Feasibility

ISCA Q27. Which of the following is not a component of Decision Support System (DSS)?
(a) User
(b) Database
(c) Model Base
(d) Artificial Intelligence (AI)

(d) Artificial Intelligence (AI)

ISCA Q30. A company is using Google Cloud Service for hosting its data / application. This use by the company of Google cloud can be classified as..
(a) Public Cloud
(b) Private Cloud
(c) Hybrid Cloud
(d) Community Cloud

(a) Public Cloud

ISCA Q37. Vouchers would be part of which system.
(a) Transaction Processing System (TPS) (b) Knowledge Management Systems (KMS)
(c) Decision Support System (DSS)
(d) Enterprise Information Systems (EIS)

(a) Transaction Processing System (TPS)

ISCA Q39. System providing information to help management to launch a new product shall be best classified as..
(a) Management Level System
(b) Operational Level System
(c) Knowledge Level System
(d) Strategic Level System

(d) Strategic Level System

ISCA Q42. SaaS model in cloud computing is a good example of..



(a) Custom Built Application

(b) Leased Application

(c) Packaged Software

(d) Bought out Application




(b) Leased Application




ISCA Q43. A security guard at the gate of ATM is an example of..

(a) Preventive Control

(b) Detective Control

(c) Corrective Control

(d) Administrative Control




(a) Preventive Control




ISCA Q46. In Database Management Systems (DBMS) concept, which of the following statement is not correct?

(a) File is a collection of Records

(b) Field is a collection of characters

(c) Characters of a collection of bits

(d) Record is a collection of files




(d) Record is a collection of files




ISCA 2 While Auditing the Input Controls in an enterprise ABC, which of the following controls will not fall under the purview of Input Controls?

(a) Source Document Controls

(b) Data Coding Controls

(c) Boundary Controls

(d) Batch Controls



ISCA 3 Which are the controls that are responsible for maintaining a chronology of the events from the time a sender dispatches a message to the time a receiver obtains the message?

(a) Boundary Controls

(b) Communication Controls

(c) Input Controls

(d) Database Controls



ISCA 4 In ABC, financial institution, the authorised officials identified existence of numerous accounts numbers of inactive accounts in their usage data of active customers. Complaint was reported to their IT team. Which type of controls can be exercised by the IT Team in the given case?

(a) Corrective Controls

(b) Detective Controls

(c) Preventive Controls



(d) Compensatory Controls




ISCA 7. Under Data Resource Management Controls, __________ controls are designed to prevent unauthorized individual from viewing, retrieving, computing or destroying the entity's data in any organization.

(a) Access

(b) Backup

(c) Concurrency



(d) Quality




ISCA 15. "Installation of Firewalls" in a telecommunication network is a classic example of ____________ controls under Information Systems' Controls.

(a) Managerial

(b) Application

(c) Preventive



(d) Corrective




ISCA 17. Under Asynchronous attacks in a telecommunication network systems, ___________ involves spying on information being transmitted over communication network.

(a) Wire-tapping

(b) Data Leakage

(c) Subversive attacks



(d) Piggybacking




ISCA 18 Provide the correct sequence of phases of Program Development Life Cycle. (1) Operations and Maintenance, (2) Coding, (3) Control, (4) Design, (5) Testing and (6) Planning

(a) (4)-(6)-(2)-(3)-(1)-(5)

(b) (6)-(3)-(4)-(2)-(5)-(1)

(c) (1)-(6)-(3)-(5)-(4)-(2)



(d) (3)-(5)-(2)-(1)-(6)-(4)




ISCA 19. In ABC, financial institution, the authorised officials identified existence of numerous accounts numbers of inactive accounts in their usage data of active customers. Complaint was reported to their IT team. Which type of controls can be exercised by the IT Team in the given case?

(a) Corrective Controls

(b) Preventive Controls

(c) Detective Controls



(d) Compensatory Controls




ISCA 22. Mr. Mysterious used to chat on Internet under the name of Ms. Ritu. He also gave her landline telephone number for further chatting. Ms. Ritu was a senior executive of the credit rating information sources of India Ltd. Mr. Mysterious was employed as financial analyst but had been forced to terminate the job earlier. Ritu started receiving mischievous calls. In the light of the given facts, state the liability of Mr. Mysterious within the purview of the Information Technology Act, 2000.

(a) Punishable for offence related to web defacement

(b) Punishable for phishing and email scam

(c) Theft of confidential information



(d) Harassment via fake public profile on social networking site.




ISCA 23. X through his computer steals the data saved in Y’s computer of valuable customers of ABC Ltd. X sold this data to the competitor of ABC Ltd. This act of X is _________.

(a) Source code theft

(b) Theft of confidential information

(c) Email Account hacking



(d) Hacking




ISCA 25. Which of the following statement is not true for Internal Control System?

(a) Facilitates the effectiveness and efficiency of operations.

(b) Helps safeguarding the assets of the entity.

(c) Assists compliance with applicable laws and regulations.



(d) Helps ensure the reliability of only Internal Financial Reporting and not External Financial Reporting.




ISCA 29. Which of the following is not a technique of Cryptography?

(a) Transposition

(b) Substitution

(c) Product Cipher



(d) Transcription




ISCA 33. Which of the following is not a type of Data Resource Management Controls under Managerial Controls?

(a) Existence Controls

(b) Concurrency Controls

(c) Production Controls



(d) Quality Controls




ISCA 34. Which of the following does not fit into best practices while dealing with passwords in order to avoid system failures?

(a) Periodic change of passwords

(b) Unlimited number of entry attempts

(c) Minimum password length



(d) Hashing of passwords




ISCA 35. Under emerging BYOD (Bring your Own Device) Threats, _______________ risk refers to the data lost from stolen or lost devices.

(a) Network

(b) Application

(c) Device



(d) Implementation




ISCA 36. In Cloud Computing, which of the following instance of Software as a Service (SaaS) allows users to explore functionality of Web services such as Google Maps, Payroll Processing, and credit card processing services.

(a) API as a Service (APIaaS)

(b) Testing as a Service (TaaS)

(c) Email as a Service (EaaS)



(d) Data as a Service (DaaS)




ISCA 37. Which activity is not incorporated during audit of Environmental Controls?

(a) Backup Power

(b) Water Detection

(c) Fire detection and Suppression



(d) Guards and Dogs




ISCA 38. As an Information Systems Auditor, mention the activity that does not form part of audit of Logical Access Controls?

(a) Access Violations

(b) Intrusion Detection and Prevention

(c) Backup Power



(d) Shared Accounts




ISCA 40. Suggest the control required to address the concerns raised with the following risk ‐ "As everybody is connected to a single system and central database, in case of failure of system, the whole business may come to stand still and may get affected badly."

(a) This can be controlled and monitored by having proper and updated backup of data as well as alternate hardware/internet arrangements.

(b) This can be controlled by removing redundant data, using techniques like data warehousing and updating hardware on a continuous basis.

(c) Access rights need to be defined carefully and to be given on "Need to know" and "Need to do" basis only.

(d) This can be controlled and minimized with the help of proper staff training system, having help manuals, having backup plans for staff turnover etc.




ISCA 41. Suggest the control required to address the concerns raised with the following risk ‐ "As data is maintained centrally, gradually the data size becomes more and more and it may reduce the speed of operation."

(a) This can be controlled and monitored by having proper and updated backup of data as well as alternate hardware/internet arrangements.

(b) This can be controlled by removing redundant data, using techniques like data warehousing and updating hardware on a continuous basis.

(c) Access rights need to be defined carefully and to be given on "Need to know" and "Need to do" basis only.



(d) This can be controlled and minimized with the help of proper staff training system, having help manuals, having backup plans for staff turnover etc.




ISCA 46. In telecommunication network, ________ is the act of following an authorized person through a secured door or electronically attaching to an authorized telecommunication link that intercepts and alters transmissions. This involves intercepting communication between the operating system and the user and modifying them or substituting new messages.

(a) Wire tapping

(b) Piggybacking

(c) Subversive attacks



(d) Data Leakage




ISCA 47. One amongst the list is a not a threat.

(a) Virus

(b) Trojan

(c) Worm



(d) Firewall




ISCA 48. Which type of risk a company becomes vulnerable to when it adopts Bring Your Own Device (BYOD)?

(a) Confidentiality Risk

(b) Device Risk

(c) Application Risk



(d) Implementation Risk




ISCA 49. Database Management Systems (DBMS) helps the organisation to do various operations on the files

(a) Adding or deleting files from database

(b) Inserting or modifying existing files

(c) Retrieving or deleting data from existing files



(d) Duplicating Files




1. ___________ is the system by which a company or enterprise is directed and controlled to achieve the objective of increasing shareholder value by enhancing economic performance.




(a) Corporate governance

(b) Business governance

(c) IT Governance

(d) Performance




Answer: (a)




2. ___________ is the system by which IT activities in a company or enterprise are directed and controlled to achieve business objectives with the ultimate objective of meeting stakeholder needs




(a) Corporate governance

(b) Business governance

(c) IT Governance

(d) Performance




Answer (c)




3. Which one is not a benefits of IT governance




(a) Increased value delivered through enterprise IT;

(b) Improved skill of employees;

(b) Improved liveliness in supporting business needs;

(d) Increased user satisfaction with IT services;




Answer (b)




4. _________ is the weakness in the system safeguards that exposes the system to threats. It may be a weakness in information system/s, cryptographic system (security systems), or other components (e.g. system security procedures, hardware design, internal controls) that could be exploited by a threat.

(a) Threat Agent

(b) Risk

(c) Vulnerability

(d) Counter measure




Answer (c)




5. ACL: ACL is an audit and data analysis tool. ACL provides powerful internal controls that identify and mitigate risk, protect profits, and accelerate performance.




6. AGILE: In this method of system development, the focus is to "Build Short and Build Often", i.e. each development effort is kept small.




7. Analytical CRM: Customer relationship management applications dealing with the analysis of customer data to provide information for improving business performance.




8. Application Program Interface (API): How one computer process (software) communicates with another. APIs may be standardized by industry agreement or government fiat, or proprietary to a specific application or vendor. The scope of the term API can vary based on its usage. It may refer to a single "call" by which one application can request information for another, the set of such calls for an application or the collection of all such application APIs used by an organization. In cloud environments, this is sometimes referred to as "Web API."




9. Application Server: A server that is used for storing applications. Users can access and use these server applications instead of loading the applications on their client machines. The application that the client runs is stored on the client. Requests are sent to the server for processing, and the results are returned to the client. In this way, little information is processed by the client, and nearly everything is done by the server.




10. Application Software Package: A set of prewritten, pre-coded application software programs that are commercially available for sale or lease.




11. Artificial Intelligence (Al): The efforts to develop computer based systems that can behave like humans, with the ability to learn languages, accomplish physical tasks, use a perceptual apparatus, and emulate human expertise and decision making.




12. Assurance Services: An objective examination of evidence for the purpose of providing an independent assessment on governance, risk management, and control processes for the organization. Examples may include financial, performance, compliance, system security, and due diligence engagements.




13. Asynchronous Transfer Mode (ATM): A networking technology that parcels information into 8-byte cells, allowing data to be transmitted between computers from different vendors at any speed.




14. Audit Hooks: Audit hooks are audit routines that flag suspicious transactions.




15. Audit Trail (or audit log): A security-relevant chronological record, set of records, or destination and source of records that provide documentary evidence of the sequence of activities that have affected at any time a specific operation, procedure, or event.




16. Authentication: The process of verifying the identity of a person or process within the guidelines of a specific authorization policy.




17. Automated Control: Controls implemented in a manner that they can work without any manual intervention.




18. Back-Office Processes: Supporting business functions such as accounting, finance, payroll, employee benefits, and IT that provide infrastructure for an organization's vision and create a platform for growth. Back-office processes are also referred to as "non-core' processes.




19. Bandwidth: The capacity of a communications channel as measured by the difference between the highest and lowest frequencies that can be transmitted by that channel.




20. BCP: A plan used by an enterprise to respond to disruption of critical business processes.




21. Blocking: A process preventing the transfer of a specified amount of funds or a specified quantity of a security.




22. Boundary Control: Controls implemented to restrict access to a system.




23. Broad Network Access: Capabilities are available over the network and accessed through standard mechanisms that promote use by heterogeneous thin or thick client platforms (e.g., mobile phones, tablets, laptops, and workstations).




24. Bugs: A software bug is an error, flaw, failure, or fault in a computer program or system that produces an incorrect or unexpected result, or causes it to behave in unintended ways.




25. Business Intelligence (BI): It refers to applications and technologies that are used to collect, provide access and analyse data and information about company’s operations.




26. Business Process Management: Methodology for revising the enterprise's business processes to use business processes as fundamental building blocks of corporate information systems.




27. Business Process Modelling: Business process design means a structured task list I process list for a business




28. Business Process Outsourcing (BPO): A process of delegating the back-office processes or non-core business functions to a third-party service provider.




29. Business Process Reengineering: The radical redesign of business processes, combining steps to cut waste and eliminating repetitive, paper-intensive tasks in order to improve cost, quality, and service, and to maximize the benefits of information technology.




30. Business Processes: The unique ways in which enterprises coordinate and organize work activities, information, and knowledge to produce a product or service.




31. Cipher Text: Information generated by an encryption algorithm to protect the plaintext and that is unintelligible to the unauthorized reader.




32. CIS: Continuous Intermittent Simulation is a module that is embedded in a data base management system, which examines all transactions that update the DBMS.




33. Cloud Computing: Cloud computing is a model for enabling ubiquitous, convenient, on-demand network access to a shared pool of configurable computing resources (e.g., networks, servers, storage, applications, and services) that can be rapidly provisioned and released with minimal management effort or service provider interaction. This cloud model is composed of five essential characteristics, three service models, and four deployment models.




34. Compliance: Adherence to policies, plans, procedures, laws, regulations, contracts, or other requirements. Compliance refers to the systems and processes that ensure conformity with business rules, policy and regulations.




35. Computer Database: A representation cif information, knowledge, facts, concepts or instructions in text, image, audio, video that are being prepared or have been prepared in a formalized manner or have been produced by a computer, computer system or computer network and are intended for use in a computer, computer system or computer network.




36. Computer Source Code: The listing of programs, computer commands, design and layout and program analysis of computer resource in any form.




37. Computer Virus: Any computer instruction, information, data or program that destroys, damages, degrades or adversely affects the performance of a computer resource or attaches itself to another computer resource and operates when a program, data or instruction is executed or some other event takes place in that computer resource.




38. Consulting Services: Advisory and related client service activities, the nature and scope of which are agreed with the client, are intended to add value and improve an organization's governance, risk management, and control processes without the internal auditor assuming management responsibility. Examples include counsel, advice, facilitation, and training.




39. Control Environment: The attitude and actions of the board and management regarding the importance of control within the organization the control environment provides the discipline and structure for the achievement of the primary objectives of the system of internal control. The control environment includes the following elements: Integrity and ethical values, Management's philosophy and operating style, Organizational structure, Assignment of authority and responsibility, Human resource policies and practices and competence of personnel.




40. Control Processes: The policies, procedures (both manual and automated), and activities that are part of a control framework, designed and operated to ensure that risks are contained within the level that an organization is willing to accept.

41. Control: Any action taken by management, the board, and other parties to manage risk and increase the likelihood that established objectives and goals will be achieved. Management plans, organizes, and directs the performance of sufficient actions to provide reasonable assurance that objectives and goals will be achieved. Control refers to the policies, procedures, practices and organization structure which are designed to provide reasonable assurance that business objectives are achieved and undesired events are prevented or detected and corrected.

42. Corporate Governance: The systems and processes, by which enterprises are directed, controlled and monitored.

43. Cryptography: The art of protecting information by transforming it (encrypting it) into an unreadable format, called cipher text. Only those who possess a secret key can decipher (or decrypt) the message into plain text.

44. Customer Relationship Management Systems: Information systems that track all the ways in which a company interacts with its customers and analyse these interactions to optimize revenue, profitability, customer satisfaction, and customer retention.

45. Customization: The modification of a software package to meet an enterprise's unique requirements without destroying the package software's integrity.

46. Cyber Forensic: An investigation method gathering digital evidences to be produced in court of law.

47. Dada Diddling: Changing data with malicious intent before or during input into the system.

48. Damage: To destroy, alter, delete, add, modify or re-arrange any computer resource by any means.

49. Data Mining: A process where data in a data warehouse is identified to discover key business trends and factors. It basically finds hidden patterns from data.

50. Data: These are the facts that are used by programs to produce useful information. Database: This is a collection of logically related records or files.

51. DBA: A Database Administrator (short form DBA) is a person responsible for the installation, configuration, upgrade, administration, monitoring and maintenance of databases in an organization.

52. Development Toots: These are CASE tools. They are used to enhance the quality of system development efforts.

53. Digital Signature: Means authentication of any electronic record by a subscriber by means of an electronic method or procedure in accordance with the provisions.

54. E-business: It is the use of the internet and other networks and information technologies to support electronic commerce, enterprise communications and collaborations and web-enabled business processes both within an internetworked enterprise, and with its customers and business partners.

55. E-commerce: The processes by which enterprises conduct business electronically with their customers and/or public at large using the Internet as the enabling technology.

56. EDI: Electronic Data Interchange (EDI) promotes a more efficient paperless environment.

57. Encryption: The process of taking an unencrypted message (plaintext), applying a mathematical function to it (Encryption algorithm with a key) and producing a cipher text.

58. End user: Anyone who uses an information system or the information it produces.

59. Enterprise Software: Set of integrated modules for applications such as sales and distribution, financial accounting, investment management, materials management, production planning, plant maintenance, and human resources that allow data to be used by multiple functions and business processes.

60. Enterprise Systems: Integrated enterprise-wide information systems that coordinate key internal processes of the firm.

61. Expert System: It is a knowledge-intensive program that solves a problem by capturing the expertise of a human in limited domains of knowledge and experience.

62. Firewall: A system or combination of systems that enforces a boundary between two or more networks, typically forming a barrier between a secure and an open environment such as the Internet.

63. Framework: Set of controls and/or guidance organized in categories, focused on a particular topic. It is a structure upon which to build strategy, achieve objectives and monitor performance.

64. Fraud: Any illegal act characterized by deceit, concealment, or violation of trust. These acts are not dependent upon the threat of violence or physical force. Frauds are perpetrated by parties and organizations to obtain money, property, or services; to avoid payment or loss of services; or to secure personal or business advantage.

65. Governance of Enterprise IT (GEIT): Concerned with IT value delivery to the business and the mitigation of IT-related risks. This is enabled by the availability and management of adequate resources and the measurement of performance to monitor progress towards the desired goals.

66. Governance: The combination of processes and structures implemented by the board to inform, direct, manage, and monitor the activities of the organization toward the achievement of its objectives. Governance ensures that stakeholder needs, conditions and options are evaluated to determine balanced, agreed-on enterprise objectives to be achieved; setting direction through prioritisation and decision making; and monitoring performance and compliance against agreed-on direction and objectives. In most enterprises, overall governance is the responsibility of the board of directors under the leadership of the chairperson. Specific governance responsibilities may be delegated to special organizational structures at an appropriate level, particularly in larger, complex enterprises.

67. Grid Computing: Where workstations on the same network have their resources pooled in order to complete computing tasks to address a single problem. Grid is sometimes used synonymously with cloud computing.

68. Hash Total: A sum obtained by adding together numbers having different meanings; the sole purpose is to ensure the correct number of data have been read by the computer.

69. IDEA: IDEA is a generalized audit software.

70. Information System: It is considered as an arrangement of a number of elements that provides effective information for decision-making and/or control of some functionalities of an organization.

71. Information Technology Controls: Controls that support business management and governance as well as provide general and technical controls over information technology infrastructures such as applications, information, infrastructure, and people.

72. Information Technology Governance: Consists of the leadership, organizational structures, and processes that ensure that the enterprise's information technology supports the organization's strategies and objectives.

73. Information: It is data that have been put into a meaningful and useful content. Input: It is the data flowing into the system from outside.

74. Internal Audit Activity: A department, division, team of consultants, or other practitioner(s) that provides independent, objective assurance and consulting services designed to add value and improve an organization's operations. The internal audit activity helps an organization accomplish its objectives by bringing a systematic, disciplined approach to evaluate and improve the effectiveness of governance, risk management and control processes.

75. Internet: It is a network of networks that consists of millions of private, public, academic, business, and government networks, of local to global scope, that are linked by a broad array of electronic, wireless and optical networking technologies.

76. Intranet: It refers to a network inside an organization that uses internet technologies such as web browsers, servers etc. to provide an internet-like environment within enterprise for information sharing, communications, collaboration and the support of business processes.

77. ISMS: An Information Security Management System (ISMS) is a set of policies concerned with information security management or IT related risks. The main objective of information security management is to implement the appropriate measurements in order to eliminate or minimize the impact that various security related threats and vulnerabilities might have on an organization.

78. ITF (Integrated Test Facilities): A testing methodology in which test data are processed in production systems.

79. Key Pair: In a symmetric crypto system, means a private key and its mathematically related public key, which are so related that the public key can verify a digital signature created by the private key.

80. Knowledge Management: The set of processes developed in an enterprise to create, gather, store, maintain, and disseminate the firm's knowledge.

81. Middleware: Software that sits between applications and operating systems, consisting of a set of services that enable interoperability in support of distributed architectures by passing data between applications. So, for Example: the data in one database can be accessed through another database.

82. Operational Level or Lower Level Management: It is the lowest level in managerial hierarchy wherein the managers coordinate the work of others who are not themselves mangers.

83. Phishing: Phishing is an attack launched to acquire information such as usernames, passwords, and credit card details (and sometimes, indirectly, money) by masquerading personification as a trustworthy entity in an electronic communication.

84. Piggybacking: Piggybacking refers to when a person tags along with another person who is authorized to gain entry into a restricted area, or pass a certain checkpoint. The act may be legal or illegal, authorized or unauthorized, depending on the circumstances.

85. RAD: Rapid Application Development is a method of system development. The key consideration is rapidity of development.

86. Return on Investment: This defines the return an entity shall earn on a particular investment (capital expenditure)

87. Risk Management: A process to identify, assess, manage, and control potential events or situations to provide reasonable assurance regarding the achievement of the organization's objectives. It refers to the culture, processes and structures that are directed to the effective management of potential opportunities and adverse effects.

Risk: The possibility of an event occurring that will have an impact on the achievement of objectives. Risk is measured in terms of impact and likelihood. Risk is the potential for an event to occur that could have an effect on the Enterprise objectives or operations.

88. Routing: The method of routing traffic through split cable facilities or duplicate cable facilities.

89. SCARF: System Control Audit Review File is a special audit technique which uses embedded audit modules to continuously monitor transaction activity and collect data on transactions with special audit significance.

90. SDLC: A method of system development. The same also referred to as linear, waterfall, traditional approach to system development.

91. Service Level Agreement (SLA): SLAs define providers' technical performance standards in terms of service agreements. In the case of cloud computing, SLAs generally address the quality of service and security protections that providers offer. According to NIST's "Draft Cloud Computing Synopsis and Recommendations," most providers assure consumers of certain standards regarding service avail ability, remedies for failure to perform, data preservation, and legal care of subscriber information, but renounce obligation for scheduled outages, force majeure events, and unauthorized modification or disclosure of subscriber data, including service in interruptions caused by malicious activity. Typically, providers also reserve the right to change the terms and pricing of their SLAs with limited advanced notice. Subscribers are typically obligated to accept certain use policies, conform to software license terms, and provide timely payments.

92. Spoofing: Spoofing is the creation of TCP/IP packets using somebody else's IP address. Routers use the "destination IP" address in order to forward packets through the Internet, but ignore the "source IP" address. That address is only used by the destination machine when it responds back to the source.

93. SRS: System Requirements Specification Document; this document is like the religious book for SDLC. This document is the output of 'Requirements Analysis' phase.

94. Steering Committee: An advisory committee usually made up of high level stakeholders and for experts who provide guidance on key issues such as company policy and objectives, budgetary control, marketing strategy, resource allocation, and decision involving large expenditures.

95. Strategic level or Top level management: It is concerned with the developing of organizational mission, objectives and strategies.

96. Structured Query Language (SQL): The SQL used by both application programmers and end users in accessing relational Databases.

97. System: A system is defined as an orderly arrangement of a set of interrelated and independent elements that operate collectively to accomplish some common purpose or goal.

98. Tactical level or middle level management: It lies in middle of management hierarchy where managers plan, organize, lead and control the activities of other managers.

99. Tally: It is an accounting Software, very user friendly even a person having the basic knowledge of accounts and computers can easily learn this package on his own.

100. Virtual Enterprise: Enterprise using networks to link people, assets and ideas to create and distribute products and services without being limited to traditional enterprise boundaries or physical location.

101. Virtualisation: A way of making better use of available hardware resources by running multiple operating systems on one server as "virtual machines", and managing the virtualized software layer separately from the hardware. With its emphasis on decoupling software from hardware, virtualization is a step on the way to cloud computing. Virtualization cannot be thought of as true cloud computing, however, because it does not offer elastic scaling of resources or automated provisioning of new virtual machine instances.


Section - A Audit


1 The basic assumption underlying the use of analytical procedures is:

(a) It helps the auditor to study relationship among elements of financial information

(b) Relationship among data exist and continue in the absence of known condition to the contrary

(c) Analytical procedures will not be able to detect unusual relationships

(d) None of the above




2. Direct confirmation procedures are performed during audit of accounts receivable balances to address the following balance sheet assertion

(a) Right and obligations

(b) Valuation

(c) Completeness

(d) Existence




3 The auditor shall express _________ opinion when the auditor, having obtained sufficient appropriate audit evidence, concludes that misstatements, individually or in the aggregate, are both material and pervasive to the financial statements

(a) Adverse

(b) Qualified

(c) Disclaimer of opinion

(d) clean




4 The agreed terms of the audit engagement shall be recorded in an audit engagement letter which shall include the following except-

(a) Responsibilities of the auditor

(b) Description of methods to be followed for obtaining audit evidence

(c) Responsibilities of management

(d) Objective and scope of the audit of the financial statements




5 The measure of the quality of audit evidence about its relevance and reliability in providing support for the conclusions on which the auditor’s opinion is based is:

(a) Sufficiency of audit evidence

(b) Appropriateness of audit evidence

(c) Accounting estimates

(d) Reasonableness of audit evidence




6 The auditor’s _________ safeguards the auditor’s ability to form an audit opinion without being affected by any influences.

(a) Objectivity

(b) Independence

(c) Confidentiality

(d) Integrity




7 Which of the following company is not exempted from reporting under CARO, 2016?

(a) Banking company.

(b) Insurance company.

(c) Company licensed to operate under section 8 of the Companies Act, 2013.

(d) Private limited company having paid up capital of Rs. 5 crore.




8 Section 144 of the Companies Act, 2013 does not excludes the statutory auditor of the company to render the services of -

(a) Investment advisory

(b) Investment banking

(c) Branch auditor

(d) Actuarial




9 As per SA 550 on Related Parties, existence of which relationship indicate the presence of control or significant influence?

(a) Friend of a family member of a person who has the authority and responsibility for planning.

(b) Holding debentures in the entity.

(c) The entity’s holding of debentures in other entities.

(d) The entity’s holding of equity in other entities.




10. When does an auditor shall modify the opinion in the auditor’s report?

(a) When, based on the audit evidence obtained, the financial statements as a whole are not free from material misstatement.

(b) When, unable to obtain sufficient appropriate audit evidence to conclude that the financial statements as a whole are free from material misstatement. © The Institute of Chartered Accountants of India

(c) (a) and (b) both.

(d) Either (a) or (b).




11 For a given level of audit risk, the acceptable level of detection risk bears ______ relationship to the assessed risks of material misstatement at the assertion level.

(a) direct.

(b) Inverse

(c) Either (a)or(b)

(d) none of the above




12 Control activities, whether within IT or manual systems, have various objectives and are applied at various organisational and functional levels. Which of the following is an example of control activities:

(a) Authorization.

(b) Performance reviews.

(c) Information processing.

(d) All of the above




13 If, as a result of a misstatement resulting from fraud, the auditor encounters exceptional circumstances that bring into question his ability to continue performing the audit, he shall-

(a) Withdraw from the engagement immediately.

(b) Report to Audit team regarding withdrawal.

(c) Determine the professional and legal responsibilities applicable in the circumstances.

(d) Ask the management for his withdrawal.




14 In order to form the opinion, the auditor shall conclude as to whether the auditor has obtained _________about whether the financial statements as a whole are free from material misstatement, whether due to fraud or error.

(a) reasonable assurance

(b) absolute assurance

(c) Limited assurance

(d) None of the above




15 When is evidential matter, generally, considered sufficient in case of stock exchange member audit?

(a) When it constitutes entire population

(b) When it is objective and relevant

(c) When it is enough to provide a basis for giving reasonable assurance regarding truthfulness

(d) When auditor collects and evaluates it independently




Answer: (b)




16 The scope of the audit of Depositories including reference to the pronouncements of the ICAI, which the auditor adheres to, generally is communicated to the client in the

i) auditor’s report

ii) engagement letter

iii) representation letter




(a) only (i)

(b) Both (i) and (ii)

(c) Both (i) and (iii)

(d) All of the above




Answer: (b)




17. Which of the following information should a successor auditor obtain during the inquiry of the predecessor auditor before accepting engagement?

i) Information about integrity of management

ii) Disagreement with management concerning auditing procedures

iii) Review of internal control system.

iv) Organisation structure




(a) (i) and (ii)

(b) (ii) and (iii)

(c) (i) , (ii) and (iii)

(d) (i) and (iii)




18. In an investigation relating to possible misappropriation of cash, the cashier says that every day the cash is counted and is reviewed by the Finance Head. Your specimen review indicates that the daily cash summary was not signed off by of the Finance Head. In this situation you should:

(a) conclude that the cashier is not telling truth

(b) consider extending investigation procedures like corroborative enquiry with the Finance Head, review of appropriate daily cash summaries etc.

(c) conclude that the Finance Head is not a responsible person

(d) conclude that daily cash summary is not relevant for the investigation




19. Current period adjustments are those adjustments that are made:

(a) only on the first occasion of the preparation and presentation of consolidated financial statements

(b) only on the first occasion of the audit of consolidated financial statements

(c) in the accounting period for which the consolidation of financial statements is done

(d) None of the above




20 Which of the following best suits the description – “The susceptibility of an assertion that could be material, either individually or in aggregate, before consideration of any related Internal Controls.”

(a) Inherent Risk

(b) Detection Risk

(c) Control Risk

(d) None of the above




21 CA. D, a chartered accountant in practice availed of a loan against his personal investments from a bank. He issued 2 cheques towards repayment of the said loan as per the instalments due. However, both the cheques were returned back by the bank with the remarks "Insufficient funds". As per Chartered Accountants Act, 1949, under which clause CA D is liable for misconduct .

(a) Clause (6) of Part I of the First Schedule

(b) Clause 2 of Part I of the Second Schedule

(c) Clause 12 of Part I of the First Schedule

(d) Clause 2 of Part IV of the First Schedule




22. As an auditor appointed under section 44AB of the Income Tax Act, 1961, under which clause of Form 3CD, you will report for amounts deemed to be profits and gains under section 32AC, 33AB or 33ABA or 33AC

(a) clause 24

(b) clause 40

(c) clauses 31

(d) clause 23




23. As per CARO, 2016, the auditor is required to report whether the company is required to be registered under section 45-IA of the Reserve Bank of India Act, 1934. If so, whether the registration has been obtained.

(a) Under Clause (xi) of paragraph 3 of the CARO, 2016,

(b) Under Clause (xvi) of paragraph 3 of the CARO, 2016,

(c) Under Clause (xv) of paragraph 3 of the CARO, 2016,

(d) Under Clause (xiv) of paragraph 3 of the CARO, 2016,




24. As per Clause (i)(c) of Paragraph 3 of the CARO, 2016, the auditor is required to report on :

(a) whether the title deeds of immovable properties are held in the name of the company. If not, provide the details thereof.

(b) whether the company has entered into any non-cash transactions with directors or persons connected with him

(c) whether any fraud by the company or any fraud on the Company by its officers or employees has been noticed or reported during the year; If yes, the nature and the amount involved is to be indicated;

(d) whether the company is maintaining proper records showing full particulars, including quantitative details and situation of fixed assets;




25. LM Ltd. had obtained a Term Loan of rupees 300 lakhs from a bank for the construction of a factory. Since there was a delay in the construction activities, the said funds were temporarily invested in short term deposits. Under which clause of CARO 2016 the auditor is required to report

(a) Under Clause (viii) of paragraph 3 of the CARO, 2016,

(b) Under Clause (xi) of paragraph 3 of the CARO, 2016,

(c) Under Clause (x) of paragraph 3 of the CARO, 2016,

(d) Under Clause (ix) of paragraph 3 of the CARO, 2016,




26.NMP Ltd is in the business of retail and has been suffering losses. The turnover of the company has been same over the last 3-5 years. The company has Oracle as its ERP package. The internal auditor of the company observed that there is no process to review the supplier master on a periodic basis to identify the cases of incorrect updation / redundant supplier codes, key fields were not made mandatory in Oracle at the time of vendor empanelment and maker checker mechanism was also not enabled in Oracle. There is no mechanism to track redundant supplier codes and block them for further transactions. For 5,750 out of 9,076 active suppliers (63.3%), no transaction had occurred in the past 180 days. For 4,972 out of these 5750, no transaction occurred in the past 1 year. For 35 out of 9,076 active suppliers, the state code in the GST Identification Number (GSTIN) updated in the supplier master did not match the state mentioned in supplier’s address. Payments valuing INR 27 crores have been made to such suppliers. Management explained that for redundant supplier codes, annual review will be conducted by the purchase team to identify such codes and, post an approval from finance, purchasing will be blocked for the respective vendors. For GSTIN and State mismatch, management has already commenced assessment to identify the reasons for such errors and all such inconsistencies will be rectified in next 6 months. Please suggest in terms of reporting.

(a) Management responses look reasonable and this matter should be dropped.

(b) The matter is more of related to hygiene and may not have any impact on the financial reporting and hence should be ignored.

(c) Internal auditor need to report this matter.

(d) Internal auditor should look at the significance of the matter. Material and on the basis of the same should decide about reporting this matter.




27. DPP Ltd is in the business of software and is in growing phase. The company’s turnover has been increasing year on year and profit margins are good. The company is also planning IPO in next 2-3 years depending on the market assessment at that point of time. It was observed by the internal auditors of the company that it does not have a documented Segregation of Duty (SOD) Matrix. Access controls were tested on basis of leading practices and following observations were identified: - Users apart from Finance & Accounts team were having access to critical financial transactions.

- Users apart from Quality department were having access to Quality Clearance transaction for raw material and finished goods.

- Multiple users having access to Purchase Order Approval though it should be confined to HODs/Purchase Heads. Management of the company explained to the auditor that the company is new and this may be required for a well established company. Please advise.

(a) Generic accounts increase the risk associated with accountability and might lead to unauthorized access which could result into impact on financials. It will also affect the transparency and auditing trail that corresponds with the account. Hence there should be a proper SOD matrix.

(b) Generic accounts increase the risk associated with accountability and might lead to unauthorized access which could result into impact on financials. It will also affect the transparency and auditing trail that corresponds with the account. There should be a process of SOD though it is not necessary to document that.

(c ) Management is right and accordingly it is not relevant for the internal auditor.

(d) Since currently the operations of the company are running smoothly, there is no need for complicating the internal business environment by setting up SOD matrix.




28. KSHTZ Ltd, listed company, is in the business of stainless steel and is more than 50 years old. The company’s turnover is INR 11000 crores and has good profit margins which have been improving over the last 2 years. The company is also planning to raise funds in another 5-6 months. The company has SAP as its ERP package. Recently there has been a change in the internal audit team. The new internal auditors observed that there have not been any approved policies and procedure in place in their audit period from 1 April 2018 to 30 September 2018. For e.g.

- Information Security Policy and Procedure

- Change Management Policy

- User Access Management Policy.




Also the policies and procedures do not have any version control, owner and review details, etc. Management of the company explained to the auditor that the company does not require this and hence this point should be ignored. Please advise.

(a) Absence of well defined and approved policies and procedures may lead to management's intended practices and objectives not being clearly communicated to and understood by organization's employees and hence there should be approved policies and procedures in place.

(b) Absence of well defined and approved policies and procedures may lead to management's intended practices and objectives not being clearly communicated to and understood by organization's employees. There should be a process to follow policies and procedures though it is not necessary to document that.

(c) Management is right and accordingly it is not relevant for the internal auditor.

(d) Absence of well defined and approved policies and procedures has not impacted the company till date and if the management has reasons not to keep this then the same should not be considered by the internal auditor.




29. ACE Pvt Ltd is a large company and has diverse operations. The company is planning to get listed to raise funds. Over the last years, the company did not use much of technology and with the changing times, the management has also identified the need to bring mechanisms in place to improve upon the use of technology. The internal auditors of the company while review of Business Continuity Planning/ Disaster Recovery Plans observed that the Identified Disaster Recovery Site of the company was in the same seismic zone as the Primary Site. Therefore the effectiveness of the Disaster Recovery (DR) Plan was not verified. The management discussed this matter with the internal audit team and explained that the present DR plan is to protect against hardware failure and building level exposure. They will plan for city level DR along with Annual Business Plan in another year. Please suggest which one of the following options is correct.

(a) DR plan is not of much relevance and should not be focused upon by the internal auditor in his report.

(b) DR plan is an outdated plan and in today’s scenario it is not required. Hence internal auditor should drop this.

(c) Absence of Disaster Recovery Site in different seismic zone might lead to failed or delayed recovery of business operations in an event of natural disaster. It is important for the management to plan this and hence internal auditor should also report this.

(d) Since the management has a plan for DR in near future this matter is not relevant to be reported.




30. ASOP Ltd is in the business of trading and manufacturing of FMCG. The turnover of the company has been increasing, however, the company has not been able to maintain its margins constant which are declining. The internal auditors of the company raised observations on the sales schemes of the company. As per the SOP, all schemes are required to be approved by the CEO of the company. However, per process it was observed that all schemes were approved by Chief Sales and Marketing Officer (CSMO). Review of sample 89 support schemes for the months of May 2018 and June 2018 highlighted that 19% (i.e. 17 schemes) were not approved by the CSMO. Management replied that there is a need for revision of SOP to reflect current paradigm. They shall amend the SOP to reflect the same. Please advise how should these matters be dealt by the internal auditors?

(a). Since the management has agreed on the observation of the internal auditor, internal auditor should drop these points.

(b) SOPs are not aligned to on-ground practices followed by concerned officials. SOPs should be updated and till then there should be a mechanism to follow the existing SOP.

(c) SOPs are not aligned to on-ground practices followed by concerned officials and the same should be reported by the internal auditor.

(d) Internal auditor should look at the materiality and basis that can ignore this as this will not have much impact.




31. BCP Ltd is in the business of manufacturing of cranes. It’s a wholly owned subsidiary of a Chinese company and follows policies and procedures of the parent company. The company’s annual turnover is INR 1000 crores. The company operates through dealers in India for making sales and pays incentives to them on the basis of delivery based schemes and other schemes which are introduced from time to time. It was observed by the internal auditors of the company that incentives amounting to INR 10 crores were paid to dealers on account of delivery based schemes for the month of October 2018. Review of cranes installations for the same period highlighted that incentive amounting to INR 30 lakhs had been paid against invalid claims. This was primarily because of absence of verification of the delivery claims with the installation data. Management replied that disbursement basis 100% verified installations has been defined as per the process. Revision in process has been done to prevent inordinate delays in reimbursements to the dealers. Please advise how should these matters be dealt by the internal auditors?

(a) Since the management has agreed on the observation of the internal auditor, internal auditor should drop these points.

(b) The impact of the matter is not significant and hence the same should be dropped.

(c) Incentive paid against non-genuine claims bear financial implications for the Company. Verified installation data should be taken for considering incentive payout. This matter should be highlighted by the internal auditor in his report.

(d). Internal auditor should ask the management to take corrective action and basis that drop this point as this is matter which is of financial implication which needs to be considered by the statutory auditors of the company.




32. PRP Ltd is a service company and is in the business of manpower consultancy. The company also has some manufacturing operations based out of Orissa. The annual turnover of the company is INR 1500 crores. The employee base of the company is very big. Please advise what internal audit procedures should be considered by the internal audit team for the audit of labour cost visà-vis wages.

i. Comparing the time booked in the booking sheets with clock cards on a sample basis.

ii. For a Piece rated wage job: a. In case of an in-process job, checking that the output booked in the booking sheet is in line with the standard output possible in the stated time. In case of a major variance, enquiring into its justification and authenticity.

b. In case of a finished job, checking the output booked in the booking sheet with the actual output generated for the period as per the production sheet. c. In case of variances, enquiring into the same.

iii. In case of a person doing more than one piece - rated job during the period, checking that: Total Time Booked – Overtime Hours = Normal Hours Available in the Period.

iv. Test checking the following with the master lists:

a. Grade booked

b. Operator code

c. Job code.

v. Average Earning Job

a. Verifying on a sample basis that the job categorised as “average earning” job does not have any piece rate as per the master file.

b. Comparing the standard time required for output booked as per the master file with the actual time booked. In case of a major variance, enquiring into its justification and authenticity.

c. Test checking the calculation of wages as per the laid down formula for arithmetical accuracy.

(a) i, ii, iii, iv and v.

(b) i, ii, iii and iv.

(c) i, iii, iv and v.

(d) i, ii, iii and v.




33. OQR Ltd is in the business of manufacturing of tractors and cranes. The company has a policy to provide after sales services to the customers in respect of its products. Please advise what internal audit procedures should be considered by the internal audit team for the audit of after sales service.

i. Assess replacement trends, nature of failures and replacement policies.

ii. Examine the percentage of replacements of manufacturing defects vis-à-vis off-take.

iii. Examine which type of products/models has a higher failure record and why.

iv. Check whether any particular dealer’s failure percentage vis-à-vis his turnover higher than the norm. If so, why.

v. Check whether there are adequate technical audit on awards of replacement.

vi. Evaluate the effectiveness of after-sales service with regard to its scope and consumer satisfaction.




Is this service prompt and timely?




(a) i, ii, iii, iv and vi.

(b) i, iii, iv, v and iv.

(c) i, ii, iii, iv and v.

(d) i, ii, iii, iv, v and vi.




34. SX Ltd is in the business of steel manufacturing having a turnover of INR 10,100 crores. The company has many plants. Each plant has a canteen and some income also gets generated in the canteen every year. Being the internal auditor what internal audit procedures may be applied to audit the canteen income?

i. Check the records maintained for the canteen operations to support all financial transactions.

ii. Review the agreements and contracts in case the canteen is run by an outside party.

iii. Compliance with laws and regulations applicable for operation of canteen - The Prevention of Food Adulteration Act & Rules, 1954, The Shops and Establishment Act, FEMA, GST, Companies Act, etc.

iv. Verify leakages that may take place, e.g., by way of non-deductions from staff or excessive consumption of food in the mess, despite fixed menus which are helpful in providing some measurement of the likely consumption of food articles. Which of the above mentioned procedures would be relevant?

(a) i, ii, iii and iv.

(b) i, ii and iii.

(c). i, ii and iv.

(d) i, iii and iv.




35.TPL Pvt Ltd is in the business of software and consultancy services. The annual turnover of the company is INR 899 crores and profits are INR 199 crores. The company is planning to get listed in the overseas market within a year. If that doesn’t happen then the company may look for funding through private placement. For some projects the company receives grants from government. These projects run upto 5-10 years. XYZ & Co LLP is the internal auditor of the company. Please advise what internal audit procedures should be considered by the internal audit team for the audit of grants received.

i. Check the donations received with the copies of receipts.

ii. Check sanction letters for any conditions attached with the donations.

iii. Examine the statements submitted for utilisation of grant.

iv. Verify the grants received from the Government or other authorities with reference to all the correspondences.

v. Verify all the bank statements of the company to trace the grants received and its utilization.




(a) i, ii, iii, iv and v.

(b) i, ii, iii and iv.

(c) i, iii, iv and v.

(d) i, ii, iii and v.




36. ONZ Ltd is in the business of trading of consumer equipments. The company’s turnover is INR 347 crores. The company has not been doing well over the last few years due to which its profitability has gone down significantly. The company charges cartage/freight from its customers. Because there is a huge cost incurred in this respect, the company ensures that this amount is recovered on time. During the performance of the internal audit procedures, the internal auditors of the company found that in some cases freight was charged in the bills manually, rather than through the automated system of generating an invoice. Internal auditor raised this point to the management. The management replied that it happens only in exceptional cases that the freight is charged manually on automated generated invoice. How would you deal with this as an auditor?

(a) Internal auditor should report this matter. © The Institute of Chartered Accountants of India

(b) Internal auditor should discuss with management about way forward and drop this point.

(c) Internal audit observation is not right.

(d) Internal auditor should ignore on the grounds of materiality.




37. MNO Ltd borrowed an amount of INR 5 crores from a financial institution during the year. The company had existing borrowings of INR 1800 crores from various banks. However, the company took loan from a financial institution for the first time. The rate of interest charged on the new loan was based on market rate of interest and there was no security for this loan. During the course of the internal audit, internal auditor could not find the borrowing agreement for the new loan and raised this point with the management. The management explained that new loan was required for a special purpose for which all other documents are available for auditor to verify – disbursement proof in the bank statements, repayments. However, the agreement was not prepared because the person who arranged the loan from financial institution was known to the company and basis verbal understanding this has been done. Please advise internal auditor.

(a) Internal auditor should report this matter as this can be a serious deficiency.

(b). Because all other proofs are available, internal auditor should ignore this point.

(c). Internal auditor should report this matter to Reserve Bank of India.

(d). Considering the insignificant amount of this new loan as compared to total borrowings of the company, this may be ignored by the internal auditor.




38. AAS Ltd is in the business of fast food chains. During the internal audit of accruals/ expenses of the company, the internal audit team observed that for some of the entries passed the narration was wrongly written as if the expense is related to the travelling expense. The vouchers were passed by the finance personnel of the company but no review mechanism was seen for this. Management explained that there is a review mechanism but this is only about narration of expenses which should not be relevant for the internal auditor. How should the internal auditor deal with this matter?

(a) The Company should perform the review of entries to check such cases and same thing should be reported by the internal auditor.

(b) The Company’s management seems reasonable here.

(c) This matter should be considered on the basis of materiality.

(d) Internal auditor should further investigate as this is indicative of fraud.




39. Medivision Industries designs and manufactures spectacles. Medivision’s year end was 31 March 2018 and its draft financial statements show a profit before tax of Rs.60 lakh. The fieldwork stage for this audit has largely been completed but there are few outstanding issues. On 1 January 2018, Medivision began the commercial production of a new range of lightweight frames which have been proven to keep their shape regardless as to how roughly they are treated. Up to 31 December 2017, the company had correctly capitalised development costs of Rs.45 lakh relating to this project. The directors believe that the new frames will have a product life of three years. The financial statements show development costs at a carrying amount of Rs.45 lakh. Medivision's accounting policy states that it amortises intangible assets on a straight-line basis. The auditor's report for Medivision is due to be signed in the next week or so, and you have been unable to resolve a disagreement with the directors concerning the amortisation of the development costs. The directors have refused to include any amortisation on the basis that sales of the product have not yet commenced. Which of the following options correctly summarises the impact on the auditor's report if the issue remains unresolved?

(a) The auditor to provide an ‘Unmodified opinion’, since the directors are correct not to include any amortisation on the basis that sales of the product have not yet commenced.

(b) The auditor to provide an ‘Unmodified opinion’ with emphasis of matter paragraph about the amortisation charge on the capitalised development costs.

(c) The auditor to provide a Modified opinion - Adverse opinion since having obtained sufficient appropriate evidence, concludes that the misstatement is both material and pervasive.

(d). The auditor to provide a Modified opinion – Qualified opinion due to material misstatement of not recording the amortization charge on the capitalised development costs, which is material but not pervasive.




40 You are an audit supervisor of Swanminathan & Associates and are currently planning the audit of your client, Zonal Co which manufactures elevators. Its year end is 31 March 2018 and the forecast profit before tax is Rs 25.26 Lakhs. At the beginning of the year, Zonal purchased a patent for Rs. 5.3 lakhs which gives them the exclusive right to manufacture specialised elevator equipment for five years. In order to finance this purchase, the entity borrowed Rs. 4.5 lakhs from the bank which is repayable over five years. Which of the following is a response to the audit risk identified by you in planning the audit for the reporting year?

(a) The audit team need to agree the purchase price to supporting documentation and to confirm the useful life is five years. Recalculate the amortisation charge to ensure the accuracy of the charge and that the intangible is correctly valued at the year end.

(b) The company has borrowed Rs.4.5 lakhs from the bank via a five-year loan. This loan needs to be correctly split between current and non-current liabilities in order to ensure correct disclosure.

(c) In accordance with Ind AS 38 Intangible Assets, the patent should be included as an intangible asset and amortised over its five-year life.

(d) Also, as the level of debt has increased, there should be additional finance costs. There is a risk that this has been omitted from the statement of profit or loss leading to understated finance costs and overstated profit.




41. Teamsg International Co is a manufacturer of electrical equipment. It has factories across the country and its customer base includes retailers as well as individuals, to whom direct sales are made through their website. The company’s year-end is 31 March 2018. You are an audit supervisor of Suraj & Co and are currently reviewing documentation of Teamsg’s internal control in preparation for the interim audit. In the past six months Teamsg has changed part of its manufacturing process and as a result some new equipment has been purchased, however, there are considerable levels of plant and equipment which are now surplus to requirement. Purchase requisitions for all new equipment have been authorised by production supervisors and little has been done to reduce the surplus of old equipment. Which of the following control can be recommended to address the internal control deficiency in the respect of the acquisition of new equipment and treatment of the old equipment.

(a). Regular review of the data on the unused equipment on the master file by a responsible official and the review to be evidenced.

(b). Supplier statement reconciliations should be performed monthly for all suppliers and these should be reviewed by a responsible official.

(c). Capital expenditure authorisation levels to be established. Production supervisors should only be able to authorise low value items, any high value items should be authorised by the board.

(d). Observe the review process by senior factory personnel, identifying the treatment of any old equipment.




42. You are a manager in the audit department of Narang & Co, and you are dealing with several ethical and professional matters raised at recent management meetings, all of which relate to audit clients of your firm: One of your client Bernwood Co has a year ending 31 March 2018. During this year, the company established a pension plan for its employees, and this year end the company will be recognising for the first time a pension deficit on the balance sheet, in accordance with Ind AS 19 Employee Benefits. The finance director of Bernwood Co has contacted the audit engagement partner, asking if your firm can provide an actuarial valuation service in respect of the amount recognised. Which of the following options needs to be considered by the audit engagement partner?

(a) The issue is whether there is a self-review threat, as the valuation of the amount recognised would be recorded in the financial statements. The audit partner should decline the work of valuation service.

(b) The issue is whether the audit firm would be likely to possess the requisite competence to provide such a valuation service. The audit partner should decline since not professionally qualified to provide the valuation service.

(c) Narang & Co. needs to assess the materiality of the figure, and the degree of subjectivity involved. If it considers that safeguards like using separate personnel, performing a second partner review, could reduce the threat to an acceptable level, then it can go ahead with both the audit and the valuation service.

(d) The audit partner could go ahead with the valuation service and disclose the fact in its audit report about the service provided during the period. This will safeguard and reduce the threat to an acceptable level.




43.PR Co. designs and manufactures specialised furniture for offices in and around the city of Mumbai. The revenue has been gradually increasing over the last few years. The main concern for PR Co is finding credit-worthy customers who will make the payment on due dates. You are assigned as the audit team member to test the controls in sales and purchase system of the entity. The year end of the entity is 31 March 2018. One of the control objectives of the sales system of PR Co is to ensure that goods and services are sold to credit-worthy customers. Which of the following control activities would assist the entity in achieving this objective?

(a) All sales orders above Rs.10 lakh is based on authorised price lists.

(b) Credit limits for all the customers are checked before sales orders are accepted.

(c) Overdue debts are chased each month by the credit controller.

(d) The aged-debt listing is reviewed by the finance director of PR Co on a monthly basis.




44. You are an audit manager of DC & Co and you are currently responsible for the audit of Beautypal Co, a company which develops and manufactures health and beauty products and distributes these to wholesale customers. Its draft profit before tax is Rs.43 lakhs and total assets are Rs.38 lakhs for the financial year ended 31 March 2018. The final audit is due to commence shortly, and the following matter has been brought to your attention: Beautypal Co has a large portfolio of property, plant and equipment (PPE). In January 2018, the company carried out a full review of all its PPE and updated the useful lives, residual values, depreciation rates and methods for many categories of asset. The finance director felt the changes were necessary to better reflect the use of the assets. This resulted in the depreciation charge of some assets changing significantly for this year. Which of the following substantive procedure should the auditor perform to obtain sufficient and appropriate audit evidence in relation to matter of depreciation on property, plant and equipment?

(a) Review the capital expenditure budgets for the next few years to assess whether the revised asset lives correspond with the planned period until replacement of the relevant asset categories.

(b) Inspect non-current asset accounts for a sample of purchases to ensure they have been properly allocated.

(c) Consider whether the proceeds on disposals of PPE are reasonable and recalculate the profit or loss disposal.

(d). For a sample of fully depreciated assets, inspect the register to ensure no further depreciation is charged.




45.As an internal auditor of LMN Bank Ltd., you have to verify the vouchers for the quarter ending 30th June 2018 of a branch at Ahmedabad. While verifying the vouchers, your team noticed that many of the bearer cheques processed by the teller have not been stamped as “paid”, when discussed with the branch manager he stated the reason as ignorance on the part of official who has been assigned the duty of verifying the vouchers. As an internal auditor, what should be your next course of action:

(a). Considering the matter as immaterial, ignore it for the internal audit report.

(b) The Branch manager should be advised to rectify the discrepancy and the observation is closed in the internal audit report noting the corrective action taken.

(c) The matter should be immediately reported to those charged with governance of LMN Bank Ltd.

(d) Report the matter in Executive summary paragraph of Internal Audit Report as it is a significant internal control lapse.




46. ALM Ltd. is a trading company engaged in the business of selling readymade garments with a turnover of around Rs. 85 crore in the year 2017-18. Your firm has been appointed as statutory auditors for the year 2018-19. In the process of audit for the half year ending 30th September, 2018 your senior has instructed you to verify the debtors of the company. While verifying the same it came to your notice that the company is not taking balance confirmations from the debtors and the balance shown in the books of company is considered final for the preparation of accounts. As a statutory auditor what should be your decision on the debtors balances:

(a) Statutory auditor should review the internal audit report and ensure as per section 143 of the Companies Act, 2013 that the company has adequate internal financial controls in place.

(b) There is no need to take debtors confirmation as it is immaterial for the purpose of Audit Report.

(c) The auditor is required to take external confirmation independently and wherever the auditor gets negative or no response or the response is doubtful an alternative audit procedure should be followed.

(d). A management representation letter should be obtained by the auditor.




47. As a Central Statutory auditor of KG Ltd. for the year 2018-19 you need to verify the bank balances for the half year ending 30th September 2018. The company is holding Bank accounts in five different banks, but you found that the bank reconciliation is not complete for some of the bank accounts. When discussed with the management they explained that the number of transactions in these accounts is very huge on daily basis and there are some old entries (existing in the reconciliation statement from the year 2008 and they are not material in nature) so it is difficult to reconcile these bank accounts. As a Central Statutory Auditor what will be your decision:

(a) The unusually old outstanding entries, as are not material in nature, should be removed from reconciliation statement and the balance in books of accounts should be considered as the balance for the balance sheet purpose.

(b) The auditor should confirm the appropriateness of the old outstanding entries by taking bank confirmations for the same to reduce audit risk and obtain a management representation letter on pending reconciliation.

(c) The auditor should disclose the matter in Notes to accounts of the audit report with respect to incomplete bank reconciliation.

(d) The auditor should communicate it to those charged with governance as deficiency in internal control.




48. You are an article assistant in PQR & Associates. You are assigned an internal audit of X Ltd., a leading company in business of dairy products. While evaluating internal controls associated with related party relationships and transactions, you come across some discrepancies. What is the basic information to be collected by you related to related party relationships and transactions? i. The identity of the entity’s related parties including changes from the prior period ii. The nature of the relationships between the entity and these related parties iii. Understanding of business activities of related parties iv. Whether the entity has entered into any transaction with these related parties during the period and, if so, the nature and extent, and the purpose of the transaction v. Materiality of related party transactions

(a) i, ii & v

(b) i, ii & iv

(c) ii, iii & iv

(d) iii, iv & v




49. AMS & Co is a computer hardware specialist and has been trading for over 6 years. The company is funded through overdrafts and loans and by several large shareholders. The financial year end is 31 March 2017. AMS had significant growth in business in previous years; however, in the current year a new competitor BOM & Co, has entered the market and through competitive pricing has gained considerable market share from AMS. One of AMS’s customers has stopped trading with them and has moved its business to BOM. In addition, a few specialist developers have left the company and joined the new company BOM. AMS has found it difficult to replace these employees due to the level of their skills and knowledge. AMS has just received notification that its main supplier who provides the company with specialist electrical equipment has ceased to trade. Which of the following audit procedures should NOT be performed in assessing whether or not AMS is a going concern?

(a) Evaluating management’s plans for the future of the business, by finding out from the financial director whether the company has gained any new customers to replace the customers lost

(b) Review board meeting minutes for evidence of progress on recruiting specialist developers to replace the ones who have left to join BOM.

(c) Analyse and discuss the entity’s last 2 years of financial statements to determine whether it is consistent with the cash flow forecast.

(d) Review the correspondence with the shareholders to assess the probability that any of the shareholders choose to increase or sell their investment




50. ASM Motor Cars co. manufactures a range of motor cars and its year end is 31 March 2018. You are the audit supervisor of Khanna & Associates and currently preparing the audit programmes for the year-end audit of ASM. The entity undertakes continuous production of cars, 24 hours a day, seven days a week. An inventory count is to the undertaken at the year end and Khanna & Associates will attend. You are responsible for the audit of work in progress (WIP) and will be part of the team attending the count as well as the final audit. WIP constitutes the partly assembled cars at the year end and this balance is likely to be material. ASM values WIP according to percentage of completion, and standard costs are then applied to these percentages. Which of the following is NOT a substantive procedure the audit could perform to obtain sufficient and appropriate audit evidence in relation to the valuation of work in progress?

(a) Discuss with management how the percentage completions are attributed to WIP

(b) Observe the procedures carried out in the count in assessing the level of WIP; consider reasonableness of the assumptions used

(c) During the count, verify all the percentage completions if they are in accordance with ASM’s policies

(d) Review the level of variances between standard and actual costs © The Institute of Chartered Accountants of India Solution




51 The audit engagement letter in case of energy audit, generally, should include a reference to each of the following except

(a) Limitations of auditing

(b) Responsibilities of management with respect to audit work

(c) Expectation of receiving a written management representation letter


(d) A description of the auditor's method of sample selection





Answer (d)




52. The auditor's permanent working paper file should normally, include

(a) Extracts from client's bank statements

(b) Past Year's Financial statements

(c) Attorney's letters

(d) Debt agreements




Answer: (a) Extracts




53 During an inventory observation an auditor may detect obsolete items by all of following except.

(a) Observing unusual amounts of rust or dust

(b) Observing items with prior year inventory tags.

(c) Computing gross profit ratios

(d) Inquiry of plants personnel




Answer (c) Reason the gross profit ratio will not detect obsolete goods in inventory although other ratios such as inventory turnover will.




54. Which of the following is not a corroborative evidence in audit of bank borrowers?

(a) Minutes of meetings

(b) Confirmations from Debtors

(c) Information gathered by auditor through observation

(d) Worksheet supporting consolidated F.St.




Answer (d)




55. In which of the following types of inventories would an auditor of a bank borrower be least likely to need the assistance of a specialist to determine existence?

(a) Baked goods

(b) Coal Pile

(c) Precious gems

(d) Art Work




Answer (a) The auditor could evaluate baked goods, but would need a surveyor for the coal pile and appraisers for the gems and art work.




56. Which of the following sections deal with qualifications of the auditor?

(a) Section 226(1) and 226(2)

(b) Section 224(1) and (2)

(c) Section 226(3) and (4)

(d) Section 224(3)(4)




Answer: (a)




57. Who among the following can be appointed as special auditor by the CG?

(a) The statutory auditor

(b) Chartered accountant in practice

(c) Any chartered accounted who is not in practice

(d) Both (a) and (b)




Answer (b)




58. If any director is disqualified from being appointed under section 274(1)(g), the special auditor should mention this fact in his audit report. For this pupose, how does he determine their eligivbility

(a) He obtains a representation from each director

(b) He obtains a management representaion

(c) He enquiries from ROC

(d) Any one of the above




Answer (a)




59. The date on auditor's report should not be ___

(a) the date of AGM

(b) later than the date on which the accounts are approved in board's meeting

(c) earlier than the date on which the accounts are approved by the manangement

(d) Both (a) and (b)




Answer (c)




60. Which of the following report not result in qualification of the auditor's opinion due to a scope limiation, in case of audit of bank borrower?

(a) Restrictions the client imposed

(b) Reliance on the report of other auditor

(c) Inability to obtain sufficient appropriate evidential matter

(d) Inadequacy of accounting records




Answer (b)




61. Which of the following is most appropriate potential reaction of the auditor to his assessment that the risk of Material misstatement due to fraud is high in relation to existence of inventory?

(a) Visit location on surprise basis to observe test counts

(b) Request inventory count at a date close to year end

(c) Vouch goods sent on approval very carefully

(d) Perform analytiacal procedures




Answer (a)




51. Which of the following statements is correct concerning the required documenttion in working papers of fraud risk assessment undertaken by the auditor?

(a) All risk factors as mentioned in SA 240, should be considered and documented along with response of them.

(b) Document the identification of fraud risk factors along with response to them.

(c) Document material fraud, risk factors and response to them

(d) No documentation is reqruired.




Answer (b)
Section B Corporate and Allied Laws (Old Course)





1. From the following information in respect of company ABC Ltd. Compute the amount the company is required to spend on account of Corporate Social Responsibility for the Financial Year 2017-2018: Financial Year Net Profit(In Cr.) 2014-15 30 2015-16 22 2016-17 27




(a) 26 Crore

(b) 52 Lacs

(c) 55 Lacs

(d) 26 Lacs




2. Mr. B, director of XRL Company from 2006. He got his DIN allotted to him on May 6, 2017. By what date he should have intimated his DIN to XRL Company?

(a) Before May 20, 2017

(b) Before June 6, 2017

(c) Before July 6, 2017

(d) Before August 6, 2017




3. Mr. Raman, is appointed as valuer in April, 2018 in ABC Ltd. He undertook the valuation of the assets of the company in 2018. In case Mr. Raman becomes interested in any property, stock etc of the company, he may be not be eligible to undertake valuation in such property of the company till:

(a) 2019

(b) 2020

(c) 2021

(d) He will never be appointed as Registered Valuer of ABC Ltd.




4. PQR Company give its assent to give guarantee to ABZ Company on the taking of loan from financial institution. According to the Companies Act, 2013, the said act should be approved by the Board of Directors. State the mode of approval adopted by the board of directors of PQR company-

(a) Board shall give approval for giving guarantee on the loan by simple majority

(b) Board shall give approval by passing circular resolution.

(c) Board shall give approval by passing resolution through special majority

(d) Board shall give unanimous approval.




5. Mr. X, a director of the company, intimated of his participation in the meeting scheduled on August, 2018. He declared his participation through electronic mode, in April 2017. State whether Mr. X is entitled to participate in the meeting to be conducted in August 2018 –

(a) Yes, intimation about such participation was made at the beginning of the calendar year

(b) No, because intimation was made in previous calendar year

(c) Yes, because company was intimated of its participation in the meeting.

(d) No, because valid period of declaration (i.e., 1 year) of his participation expired.




6. In compliance to the Companies Act, 2013, at least one woman director shall be on Board of such class or classes of companies as may be prescribed. Ms. Riya is keen to hold the office of woman director in a company. She has selected some companies in which there is a vacancy for the woman director. Advice Ms. Riya in selecting the companies which are mandatorily required to appoint a woman director:

(a) PQR Limited which is a unlisted company and having paid up share capital of 150 crore rupees as per the last date of latest audited financial statements.

(b) ABC Limited which is a listed company and having a turnover of 150 crore rupees as per the last date of latest audited financial statements.

(c) XYZ Limited which is a unlisted company and having a turnover of 350 crore rupees as per the last date of latest audited financial statements.

(d) Both in ABC Limited and XYZ Limited




7. A director of XYZ, a Pvt. Ltd. takes a loan from its company. Due to some reasons, he fails to repay the debt within the given time period. He request board of directors to give him time for repayment of debt. State which of the below statements is correct with respect to the exercise of the power in the given situation as per the Companies Act, 2013-

(a) Power to fix the time limit for repayment of any debt due from director can be exercised only by members by special resolution at a general meeting.

(b) Power to fix the time limit for repayment of any debt due from director can be exercised by Board of the company itself.

(c) Power to fix the time limit for repayment of any debt due from director can be exercised with the prior permission of the company in general meeting while taking debt.

(d) Board shall not exercise this power if the provision related to repayment of debt is contained in the articles of the company.




8. Under what circumstances the meeting of the creditors may be dispensed by the NCLT?

(a) if 70% of the creditors in value agree and confirm to the scheme by way of affidavit

(b) if 80% of the creditors in value agree and confirm to the scheme by way of affidavit

(c) if 90% of the creditors in value agree and confirm to the scheme by way of affidavit

(d) None of the above




9. When can an application be made to Tribunal for constitution of a winding up committee to assist and monitor the progress of liquidation proceedings by the Company Liquidator in carrying out the function?

(a) Within two weeks from the date of passing of winding up order

(b) Within three weeks from the date of passing of winding up order

(c) Within four weeks from the date of passing of winding up order

(d) Within six weeks from the date of passing of winding up order




10. A Ltd, appointed Mr. A & Mr. B as directors of the Company, by passing of single resolution for election of these two. Later on it came in the notice of the Company that there might be some confusion regarding the compliance of the applicable provisions. State the correct statement in the light of the Companies Act, 2013 as to the an appointment and to the validity of acts of said appointed directors ?

(a) Void Appointment & acts of the directors are void ab-inito.

(b) Void Appointment & acts of the directors are valid until defect in the appointment is shown to the Company.

(c) Valid Appointment & so acts are also valid.

(d) Appointment is valid subject to ratification by shareholder in general meeting.




11. Mr. Mahesh returned from abroad, was left unspent with the foreign currency USD 1,000.This amount can be retained with him –

(a) for 60 days

(b) for 90 days

(c) for 120 days

(d) for 180 days




12. RAB Bank Limited, a banking company, has defaulted in the payment of dues to their catering contractor. Can the contractor, as an operational creditor initiate insolvency process against the bank-

(a) Yes, operational creditors are entitled

(b) No, financial service providers are excluded

(c) Yes, banking companies are covered under this code

(d) No, catering is an excluded service under the Code




13. The time line of 180 days for the Corporate Insolvency Resolution process commences from the

(a) Date of Debt

(b) Date of preferring the application

(c) Date of admission of application by NCLT

(d) 90 days after the debt is due




14. ABC and Co, the tax consultants of X Limited, for which an interim resolution professional – Mr A, has been appointed under the Corporate Insolvency resolution process has refused to furnish information to Mr A on the grounds of client confidentiality. Are they right

(a) Yes, they are right

(b) No, the Code provides powers to the IRP to access all information from various parties

(c) Partly right, they can do so only after consent of the directors

(d) Mr A is not right in even asking for this information




15. Operational creditors are entitled to receive notice of meetings of Committee of creditors if their aggregate dues are not less than …. % of the total debts of the corporate debtor

(a) 20%

(b) 10%

(c) 15%

(d) 5%




16. According to the Companies Act, 2013, the draft minutes of a Board meeting held through audio visual means shall be circulated among all the directors within ………. Of the meeting:

(a) 10 days

(b) 15 days

(c) 30 days

(d) One month




17. Minimum threshold prescribed for applicability of SARFAESI Act on NBFCs is –

(a) 1 crore

(b) 10 crore

(c) 100 crore

(d) 500 crore




18. Drug trafficking is a punishable offence in India. Suppose, Mr. X & Mr. Y, are involved in drug trafficking including imported drugs trafficking. Under which Act, Mr. X & Mr. Y can be prosecuted?

(a) Narcotic Drugs and Psychotropic Substances (NDPS) Act, 1985

(b) Prevention of Money Laundering Act, 2002

(c) Foreign Exchange Management Act, 1999

(d) Offences under the Unlawful Activities (Prevention) Act, 1967




19. Ruby Ltd. filed an application to the NCLT stating that corporate insolvency resolution process against him, cannot be completed within the 90 days under the fast track insolvency resolution process. Considering application and on being satisfied , NCLT ordered to extend the period of such process by 30 days. Later, again Ruby Ltd. initiated an application for further extension of time period of insolvency process by 15 days. Decide in the given situation, whether NCLT, can extend timelines by further 15 days.

(a) Yes, because extension of duration in toto, is not exceeding 45 days.

(b) Yes , depends of the facts , if it is justified , NCLT may extend the timelines.

(c) No, extension of the fast track insolvency resolution process shall not granted more than once.

(d) (a) & (b)




20. KDS Agro Pvt. Ltd., a newly incorporated company has not mentioned the names of first directors of the company in its Articles of Association. Referring the provisions of the Companies Act, 2013, who shall be deemed to be the first directors of the company?

(a) The members of the company shall be deemed to be the first directors of the company.

(b) The subscribers of the company shall be deemed to be the first directors.

(c) None shall be deemed to be the first directors of the company.

(d) The shareholders shall appoint first directors in the General Meeting.




21. The turnover of XYZ Ltd. as on the last date of latest audited financial statements is 400 crore rupees. An Intermittent vacancy of the women director arises on 15th June, 2018 in the company. The immediate Board meeting was held on 14th October, 2018. The vacancy of the women director shall be filled up by-------:

(a) Appointment of women director is not mandatory

(b) 14th August, 2018

(c) 14th September, 2018

(d) 14th October, 2018




22. In compliance to the Companies Act, 2013, at least one woman director shall be on Board of such class or classes of companies as may be prescribed. Ms. Riya is keen to hold the office of woman director in a company. She has selected some companies in which there is a vacancy for the woman director. Advice Ms. Riya in selecting the companies which are mandatorily required to appoint a woman director:

(a) PQR Limited which is a unlisted company and having paid up share capital of one 50 crore rupees as per the last date of latest audited financial statements.

(b) ABC Limited which is a listed company and having a turnover of 150 crore rupees as per the last date of latest audited financial statements.

(c) XYZ Limited which is a unlisted company and having a turnover of 350 crore rupees as per the last date of latest audited financial statements

(d) Both in ABC Limited and XYZ Limited




23. XYZ Limited is an unlisted company engaged in manufacturing of fabrics. The turnover of the company as on the last date of latest audited financial statements is Rs. 100 Crore. The company is having 7 directors in its Audit Committee. Advice the company by choosing the correct option for the company regarding requirement of appointment of independent director and minimum number of independent directors if required?

(a) The appointment of independent directors is mandatorily required under XYZ Limited and the minimum number of independent directors is two.

(b) The appointment of independent directors is mandatorily required under XYZ Limited and the minimum number of independent directors is three.

(c) The appointment of independent directors is mandatorily required under XYZ Limited and the minimum number of independent directors is four.

(d) The appointment of independent directors is not mandatorily required under XYZ Limited.




24. Amar Textiles Limited, a listed company, engaged in the production of furniture and fittings in Pune. The company is having 50,000 small shareholders. The small shareholders wanted to elect a small shareholders’ directors amongst themselves so that their issues are resolved during the Board meetings at the earliest. 500 small shareholders served a notice for appointment of a small shareholder in the Board. Decide the validity of the notice by the small shareholders:

(a) Notice by 500 small shareholders is valid and company may appoint a small shareholder director.

(b) Notice by 500 small shareholders is not valid and company may appoint a small shareholder ‘director on the requisition of 1000 small shareholders.

(c) Notice by 500 small shareholders is not valid and company may appoint a small shareholder director on the requisition of 5000 small shareholders.

(d) Small shareholders director cannot be appointed in the company as the company doesn’t fulfil the condition for appointment of small shareholders’ director.




25. Stylish Technology Limited engaged in the manufacturing of mobiles and chargers. The company’s Board of Directors consist of 8 directors i.e. Mr. Ram (Director), Mr. Shyam (Director), Mr. Mohan (Director), Mr. Vijay (Director), Mr. Naresh (Director), Mr. Ashish (Independent Director), Mr. Neeraj (Independent Director) and Mr. Anil (Small shareholders’ director). Calculate the number of directors to retire at Annual General Meeting held on 15th September, 2018.

(a) One

(b) Two

(c) Three

(d) Four




26. Prince Ltd. desires to appoint an additional director on its Board of directors. The Articles of the company confer upon the Board to exercise the power to appoint such a director. As such M is appointed as an additional director on 12th December, 2017. The Annual General Meeting ought to have been held on 17th August, 2018 but adjourned to 5th October, 2018. Decide the date upto which M can continue as director in Prince Ltd.?

(a) 17th August, 2018

(b) 30th September, 2018

(c) 5th October, 2018

(d) The appointment of additional director is not valid.




27. Mr. Q, a Director of PQR Limited proceeding on a foreign tour for six months, appointed Mr. Y as an alternate director to act for him during his absence. The articles of the company provide for © The Institute of Chartered Accountants of India appointment of alternate directors. Mr. Q claims that he has a right to appoint alternate director. Examine the validity of Mr. Q claim.

(a) Claim by Mr. Q is valid as the Articles provide for appointment of alternate directors

(b) Claim by Mr. Q is not valid as the authority to appoint alternate director has been vested in the board of directors only and that too subject to empowerment by the Articles

(c) Mr. Y cannot be appointed as alternate director as Mr. Q proceeds on a foreign tour for six months only

(d) None of the above




28. XYZ Limited is a listed public company having a paid-up capital of twenty crore rupees as on 31st March, 2017 and a turnover of one hundred fifty crore rupees during the year ended 31st March, 2017. The total number of directors is thirteen. State the minimum number of independent directors that the company should appoint.

(a) 2

(b) 3

(c) 4

(d) 5




29. Seafood Limited, a public limited company was incorporated on 1st April, 2016. The company has conducted four Board Meetings during the financial year 2016-17 i.e. on 6th April, 2016, 28th August, 2016, 30th September, 2016 and 30th March, 2017. Decide on the validity of the frequency of the Board Meeting:

(a) There is no contravention of the provision related to holding of board meeting as 4 board meetings has been held during the year.

(b) There is no contravention of the provision related to holding of board meeting as the first board meeting was held within 30 days of incorporation.

(c) There is a contravention in respect of the conduct of the board meetings as gap between two consecutive board meetings (6th April and 28th August) is 143 days and gap between two consecutive board meetings (30th September and 30th March) is 181 days.

(d) There is a contravention in respect of the conduct of the board meetings as gap between two consecutive board meetings (6th April and 28th August) is 123 days and gap between two consecutive board meetings (30th September and 30th March) is 141 days.




30. There are 9 directors in a company and out of which 2 officers of the directors have fallen vacant. What will be the quorum for the board meeting?

(a) 2

(b) 3

(c) 4

(d) 5




31. The Board of Directors of ABV Limited is desirous to delegate some of its powers to its committee of directors as they find it difficult to manage everything on their own. The Board of Directors are confused between the various powers of the Board as which power can be delegated or not. Advice the Board amongst the following powers which they can delegate to its committee of directors:

(a) To authorise buy-back of securities

(b) To invest the funds of the company

(c) To diversify the business of the company

(d) To take over a company or acquire a controlling or substantial stake in another company




32. The Board of directors of Very Well Ltd., wants to contribute Rs. 60,000 to a charitable organization during the financial year 2017-2018. During the financial year 2015-2016, the company suffered losses. The directors are contemplating to contribute the said amount in spite of the losses. In this connection, state whether the directors can do so?

(a) No, Very Well Ltd. cannot contribute Rs. 60,000 in 2017-2018 as the company suffered losses in 2015-2016.

(b) No, the board of directors are not authorized to contribute to bonafide charitable and other funds.

(c) No, Permission of the company in general meeting shall be required for contribution to bonafide charitable and other funds

(d) Yes, Very Well Ltd. Can contribute Rs. 60,000 to a charitable organization inspite of losses in 2015- 2016 provided it is to a bonafide charitable fund.




33. A financial creditor, FC through an assignment agreement, assigned here the debt to the X trust. X trust filed the petition for initiation of corporate Insolvency resolution process (CIRP) against Corporate debtor, CR. State the correct statement with respect to the competency of the X trust in the filing of the petition in the above situation-

(a) X Trust is not a competent applicant as per section 6 of the IBC, 2016

(b) X Trust is being authorized by the FC to file an application

(c) X Trust in the capacity of financial creditor can file a valid petition.

(d) None of the above




34. As per the Insolvency & Bankruptcy Code, 2016, resolution plan is prepared by ----------- is submitted to -----------------

(a) Committee of Creditors, Adjudicating Authority

(b) Resolution applicant, committee of creditors

(c) Resolution applicant, Resolution Professional

(d) Committee of Creditors, Resolution Professional




35. Committee of creditors of Corporate debtors was constituted on 17.3.2018. Time limit, within which the first meeting of committee of creditors should be held, is ----------------.

(a) 20.3.2018

(b) 22.3.2018

(c) 24.3.2018

(d) 31.3.2018




36. Raghav plan to visits USA. He can carry foreign currency in cash for travel abroad-

(a) USD 2000

(b) USD 3000

(c) USD 5000

(d) USD 10,000




37. Capable Limited appoints Mr. Vikas as the Managing director of the company. The board of directors entrusted him with some powers. Mr. Vikas is not ready to do such administrative acts authorised by the Board of Directors keeping in view that he shall be entrusted with substantial powers of the management. Decide the acts which Mr. Vikas can undertake:

(a) To draw and endorse any cheque on the account of the company in any bank

(b) To sign the financial statements of the company

(c) To draw and endorse any negotiable instrument

(d) to Sign any certificate of share




38. Mr. X was appointed as Managing Director for life by the Articles of Association of a private company incorporated on 1st June, 2018. Examine in this connection, Can Mr. X be appointed for life as Managing Director?

(a) Yes, Mr. X can be appointed as managing director for life in a private company

(b) No, Mr. X cannot be appointed as managing director for life in private company as only public companies are allowed to appoint managing director for life

(c) No, Mr. X cannot be appointed as managing director for life in private company as term of managing director cannot exceed five years at a time

(d) No, Mr. X cannot be appointed as managing director for life in private company as private companies are not allowed to appoint managing director




39. Green Farms Private Limited was incorporated on 12th December 2013. Ms. Nidhi Shah, Chartered Accountant, Pune was proposed to be appointed as their first auditor through proposal letter dated 16th December 2013. The Board has requested her to give consent to act as Auditor of the Company. However the Auditor has failed to give her consent till 16th January 2014 and hence the auditor cannot be appointed by the Board. Kindly suggest a way forward for appointment of first auditor of the Company from the following options:

a) Company shall seek approval from Central Government for appointment of first auditor;

b) Company shall appoint the first Auditor in the subsequent Board Meeting of Directors;

c) Board shall inform the members who shall at an extra ordinary general meeting to appoint first auditor within prescribed time;

d) None of the above.




40. XP Ltd declared 12% dividend to its Equity Shareholders. However, Company missed to transfer unpaid dividend to bank account even after 40 days from declaration of Dividend. In such case how much interest will be payable?

a) 8% p.a.

b) 16% p.a.

c) 10% p.a.

d) 12% p.a.




41. Mr. Narayan Shastri was appointed as an Additional Director of XYZ Limited in July, 2015. Immediately after his appointment in July, 2015, on behalf of the Company he entered into an agreement with NY Private Limited for supplies of raw material. Later he was regularized as a Director in ensuing annual general meeting. In 2016 he signed Contract with Laxmi vendors. At the end of year, in December 2016, management came to know that his appointment was not valid as he was disqualified to act as a Director of any Company. He signed one more agreement in January 2017 with Saraswati vendors. In such scenario, what will be the status of contract/agreements he signed on behalf of XYZ Limited?

a) All agreement/ contracts will become invalid;

b) All agreement/ contracts will be valid;

c) All agreement/ contracts before December 2016 will be valid;

d) All agreement/ contracts before December 2016 will be invalid;




42. Mr. N, Managing Director resigned w.e.f. 13th March, 2014 due to preoccupation. Mr. M, Whole Time Director and Mr. O, Director, resigned w.e.f. 20th March, 2015 as a part of reconstruction of the company. Mr. M and Mr. O made an application to the Board for compensation for loss of office. When Mr. N came to know that he also asked for compensation. Who will be eligible for such compensation?

a) Mr. N;

b) Mr. M;

c) Mr. N & Mr. M;

d) Mr. M & Mr. O;




43. Mr. Rajesh Jathar and Mr. Veena Jathar are the shareholders of NY Private Limited. Mr. Jathar is out of Country for business purpose. They have to have a Board meeting through video conference to comply with the requirements. Which of the following items they cannot discuss in such meeting?

a) Convening of General meeting.

b) Approval of Board’s report.

c) Appointment of Managing Director.

d) Transfer of Shares.




44. PQR Limited and LMN Limited have proposed Scheme of Amalgamation between them under Section 232 of the Companies Act 2013. They are seeking your advice on which of the following approvals can be asked for in the petition to be filed before NCLT for the proposed scheme. The Petition for proposed Scheme of Amalgamation can seek approval for –

a) Change in Main Object Clause of Memorandum of Association;

b) Reduction of Share Capital;

c) Dissolution of the Transferor Company without winding up;

d) All of the above.




45. Members of Agricultural Co-operative Society are now planning to form a Producer Company under Part IX A of the Companies Act, 1956 now known as Companies Act 2013. Kindly suggest which of the following combination can form a Producer Company under this act –

a) Ten or more producers (individuals);

b) Five or more producer institutions;

c) Combination of above two;

d) None of the above.




46. You are a Whole Time Director of Choco-chips Private Limited who wishes to appoint Mr. Vanilla Sequera as its Managing Director who has attained the age of 72 years. However, the Board has got to know about the fact that no company shall appoint or continue the employment of any person as managing director, whole-time director or manager who is below the age of twenty-one years or has attained the age of seventy years. You are requested by the board to evaluate the situation and suggest on whether he can be appointed as Managing Director?

a) Yes, he can be appointed as MD by the Board of Directors;

b) No, he cannot be appointed as MD at all;

c) Yes, he can be appointed as MD by shareholders through Special Resolution;

d) Yes, he can be appointed as MD by making an application to the NCLT.




47. ABP Ltd. is a Company having paid up Capital of INR 100,000,000. It needs to appoint key managerial personnel. It can use combination of:

i) Chief Executive officer, Company Secretary and Manager

ii) Chief Executive officer, Company Secretary and Chief Financial officer

iii) Managing Director, Company Secretary and Manager

iv) Managing Director, Company Secretary and Chief Executive officer Which options are available for the Company?

a) (i) (ii) & (iii)

b) (ii) (iii) & (iv)

c) (i) (ii) & (iv)

d) (i) (iii) & (iv)




48. Mr. A was appointed as a Manager of PQR Ltd for the period of five years on 20th June, 2015. Considering his performance and dedication, before completion of his tenure, management decided to re-appoint him as a manager. On which date his re-appointment will be considered valid?

a) 24th June 2019

b) 1st February 2019

c) 12th March 2018

d) 10th September 2018




49. Mr. M is a businessman and he owns number of companies. As of now, he is Director in 21 Companies. As his number of Directorship contravenes the provision of the Act, his consultant have asked him to resign from 2 Companies. However there is one more condition regarding number of Directorship in Public Companies. What is that Condition?

a) maximum number of public companies in which a person can be appointed as a director shall not exceed Ten;

b) maximum number of public companies in which a person can be appointed as a director shall not exceed Five;

c) maximum number of public companies in which a person can be appointed as a director shall not exceed Fifteen;

d) maximum number of public companies in which a person can be appointed as a director shall not exceed Twenty;




50. Mr. Nagarjuna decided to resign from MGT Private Limited due to preoccupation. He sent his resignation letter dated 12th June, 2017 to the Company stating that he will resign w.e.f. 15th June, 2017. Due to nonreceipt of any communication from the Company he dropped a mail on 17th June, 2017, to confirm whether Company has received his letter. Finally, company received his letter on 25th June, 2017. In this case, from which date his resignation will be effective?

a) 12th June, 2017

b) 15th June, 2017

c) 17th June, 2017

d) 25th June, 2017




Paper 7 Direct Tax Laws & International Taxation (New Course)

1. Air India Ltd. has paid amount of Rs.20 lakhs during the year ended 31.3.2019 to Airports Authority of India towards landing and parking charges.

(a) No tax is deductible at source from such payment

(b) Tax is deductible at source@2% u/s 194C on such payment

(c) Tax is deductible at source@2% u/s 194-I on such payment

(d) Tax is deductible at source@10% u/s 194-I on such payment




2. A Ltd. credited Rs.28,000 towards fees for professional services and Rs.27,000 towards fees for technical services to the account of Ram in its books of account on 12.11.2018. The total sum of Rs.55,000 was paid by cheque to Ram on the same date.

(a) No tax is deductible at source from such payment

(b) Tax is deductible at source@10% u/s 194J on the entire payment of Rs.55,000

(c) Tax is deductible at source@10% u/s 194J on Rs.25,000 (Rs.55,000 - Rs.30,000)

(d) Tax is deductible at source@2% u/s 194-J on Rs.25,000 (Rs.55,000 - Rs.30,000)




3. Mr. Hari, a salaried individual, pays rent of Rs.55,000 per month to Mr. Raghav from June, 2018. Which of the following statement is correct?

(a) No tax is required to be deducted for F.Y.2018-19 since Mr. Hari is not subject to tax audit u/s 44AB

(b) Mr. Hari has to deduct tax@5% from rent paid every month

(c) Mr. Hari has to deduct tax@5% on the entire rent paid for F.Y.2018-19 from the rent payable for March, 2019

(d) Mr. Hari has to deduct tax of Rs.55,000 from rent payable for March, 2019




4. A Ltd., an Indian company, has a wholly owned subsidiary in Sri Lanka, and it extends corporate guarantee to the said non-resident subsidiary. If the amount guaranteed is Rs.90 crore, the Assessing Officer has to accept the guarantee fee declared by A Ltd. for F.Y.2018-19, if the guarantee fee declared is –

(a) Rs.45 lakhs

(b) Rs.80 lakhs

(c) Rs.90 lakhs

(d) Either (a) or (b)




5. ABC Ltd. an Indian company paid dividend distribution tax under section 115-O in respect of dividend distributed by it to its resident and non-resident shareholders. Mr. John, a shareholder of ABC Ltd. and a resident of Country X, has to pay tax in Country X on dividend received by him from ABC Ltd., as per the domestic tax laws of Country X. This is an example of:

(a) Juridical double taxation (b) economic double taxation (c) territorial double taxation (d) municipal double taxation




6. Music Academy, as per its rules, pays a fixed honorarium per concert to each musician performing in the concerts organised by it. Hari, a violinist, however, refuses to accept this sum. If he requests Music Academy to pay such sum directly to Aid Us, an unregistered institution providing relief to the poor and needy in rural India, what would be the tax consequence? (a) No amount would be chargeable to tax in the hands of Mr. Hari, since this is a case of diversion of income at source by overriding title. (b) The amount payable to Aid Us would be chargeable to tax only in the hands of Mr. Hari, since it is a case of application of income (c) The amount payable to Aid Us would be chargeable to tax only in the hands of the institution which has received the amount (d) The amount payable to Aid Us would be chargeable to tax both in the hands of Mr. Hari and in the hands of the institution.




7. An application for advance ruling was made on 31.05.2018 in relation to a transaction proposed to be undertaken by Mr. Andrew, a resident of Germany. On 07.07.2018, he decides to withdraw the said application. (a) Application cannot be withdrawn once filed (b) Application can be withdrawn on 07.07.2018 only with special permission of Principal Chief Commissioner (c) Application cannot be withdrawn since 30 days from date of application have passed (d) Application can be withdrawn on 07.07.2018 with permission of the AAR, if the circumstances of the case so justify




8. X Ltd., a company engaged in the business of manufacturing, paid Rs.2 lakh to IISc, Bangalore (an approved and notified institution) for scientific research. It also incurred capital expenditure of Rs.12 lakh (including cost of acquisition of land Rs.5 lakh) on in-house research and development facility as approved by the prescribed authority. The deduction under section 35 for A.Y.2019-20 would be (a) Rs.9,00,000 (b) Rs.13,50,000 (c) Rs.14,00,000 (d) Rs.10,00,000




9. Mr. X, set up a manufacturing unit in Warangal in the state of Telangana on 01.06.2018. It invested Rs.30 crore in new plant and machinery on 1.6.2018. Further, it invested Rs.25 crore in the plant and machinery on 01.11.2018, out of which Rs. 5 crore was second hand plant and machinery. The depreciation allowable under section 32 for A.Y.2019-20 is (a) Rs.15.375 crore (b) Rs.20.375 crore (c) Rs.14.875 crore (d) Rs.11.375 crore




10. Y Ltd. purchased computers of the value of Rs.10 lakhs in November, 2018 and installed the same in its office. The depreciation allowable under section 32 for A.Y.2019-20 is respect of the same is – (a) Rs.6 lakhs (b) Rs.3 lakhs (c) Rs.4 lakhs (d) Rs.2 lakhs




11. Mr. Arvind, engaged in the business of wholesale trade, has a turnover of Rs.90 lakhs for P.Y.2017-18 and Rs.210 lakhs for P.Y.2018-19. In the P.Y.2018-19, he paid salary of Rs.3 lakhs to Mr. Hari, a resident, without deduction of tax at source and commission of Rs.25,000 to Mr. Rajesh, a resident, without deduction of tax at source. The disallowance under section 40(a)(ia) while computing business income of A.Y.2019-20 would be – (a) Rs.3,25,000 (b) Rs.97,500 (c) Rs.90,000 (d) Nil




12. A public charitable trust registered under section 12AA for the previous year ended 31.3.2019, derived income of Rs.10 lakhs from properties held by trust and voluntary contributions from public 15 lakhs, out of which Rs.8 lakhs was applied for charitable purposes and Rs.4 lakhs towards repayment of loan taken for construction of orphanage. The total income of the trust for A.Y.2019-20 is – (a) Rs.13 lakhs (b) Rs.9.25 lakhs (c) Rs.13.25 lakhs (d) Rs.17 lakhs




13. If Country A is a notified jurisdictional area (NJA), then, the rate at which interest receivable from a infrastructure debt fund notified u/s 10(47) is taxable in the hands of Mr. Ram, a resident of Country A, and the rate at which tax has to be deducted at source on such income are, respectively, - (a) 30% and 5% (b) 5% and 5% (c) 30% and 30% (d) 5% and 30%




14. In October, 2014, Mr. Raghav, an Indian citizen who is a non-resident, bought 500 Global Depository Receipts (GDRs) of Alpha Limited, India, issued in accordance with the notified scheme of the Central Government against the company’s initial issue of shares in foreign currency. In January, 2019, he sold 300 GDRs outside India to Mr. Joe, a citizen and resident of a country outside India and 200 GDRs to Mr. Kamal, a Resident but not ordinarily resident in India. What are the tax consequences of such sale transaction under the Incometax Act, 1961?

(a) Capital gains arising on sale of 500 GDRs shall be subject to tax @20% with indexation benefit in India

(b) No capital gains would arise on sale of 500 GDRs in India, since the GDRs are purchased in foreign currency

(c) No capital gains would arise on sale of 300 GDRs, but capital gains arising on sale of 200 GDRs shall be taxed in India @10% without indexation benefit

(d) No capital gains would arise on sale of 300 GDRs, but capital gains arising on sale of 200 GDRs shall be taxed @20% with indexation benefit in India




15. If ABC Ltd. has two Units, Unit 1 is engaged in power generation business and Unit 2 is engaged in manufacture of wires. Both the units were set up in Karnataka in the year 2014. In the year 2018-19, twenty lakh metres of wire are transferred from Unit 2 to Unit 1 at Rs.125 per metre when the market price per metre was Rs.180. Which of the following statements is correct?

(a) Transfer pricing provisions would be attracted in this case

(b) Transfer pricing provisions would not be attracted in this case since Unit 1 and Unit 2 belong to the same company and are not associated enterprises.

(c) Transfer pricing provisions would not be attracted in this case as it is not an international transaction since both Units are in India. However, for the purpose of Chapter VIA deduction, the profits of power generation business shall, however, be computed as if the transfer has been made at the market value of Rs.180 per MT.

(d) Transfer pricing provisions would not be attracted in this case due to reasons mentioned in both (b) and (c) above.




16. Which of the following is not an eligible international transaction for application of safe harbor rules?

(i) Preparation of user documentation

(ii) Receipt of intra-group loans where the amount of loan is denominated in Indian rupees

(iii) Providing implicit corporate guarantee

(iv) Purchase and export of core auto components

(v) Receipt of intra-group services from group member Choose the correct option




(a) Only (ii)

(b) (ii) & (v)

(c) (ii), (iv) & (v)

(d) (ii), (iii), (iv) & (v)




17. XYZ Ltd. has failed to report an international transaction entered by it with PQR Inc., which is a specified foreign company in relation to XYZ Ltd. What would be the penalty leviable in this case?

(i) 2% of the value of transaction

(ii) 50% of tax payable on under-reported income

(iii) 200% of tax payable on under-reported income




Choose the correct option

(a) Only (i)

(b) Only (iii)

(c) (i) & (ii)

(d) (i) & (iii)




18. Alpha Ltd.’s total income of A.Y.2019-20 has increased by Rs.34 lakhs due to application of arm’s length price by the Assessing Officer on transactions of purchase of goods from its foreign holding company in respect of a retail trade business carried on by it, and the same has been accepted by Alpha Ltd., then, -

(a) business loss of A.Y.2015-16 cannot be set-off against the enhanced income

(b) deductions under Chapter VI-A cannot be claimed in respect of the enhanced income

(c) unabsorbed depreciation of A.Y.2010-11 cannot be set-off against the enhanced income

(d) Business loss referred to in (a), deductions referred to in (b) and unabsorbed depreciation referred to in (c) cannot be set-off against the enhanced income.




19. Mr. Anjan, a property dealer, sold a flat in Mumbai, the stamp duty of which is Rs.2 crores for Rs.1.80 crores to his friend Mr. Ashwin, a college lecturer. Mr. Anjan had purchased the flat one year back for Rs.1.50 crores and the stamp duty value on that date was also Rs.1.50 crores. What are the tax implications of such sale?

(a) Rs.50 lakhs would be taxable as short-term capital gains in the hands of Mr. Anjan. There would be no tax implication in the hands of Mr. Ashwin

(b) Rs.50 lakhs would be taxable as business income in the hands of Mr. Anjan. There would be no tax implication in the hands of Mr. Ashwin

(c) Rs.50 lakhs would be taxable as business income in the hands of Mr. Anjan and Rs.20 lakhs would be taxable as income from other sources in the hands of Mr. Ashwin.

(d) Rs.50 lakhs would be taxable as short-term capital gains in the hands of Mr. Anjan and Rs.20 lakhs would be taxable as income from other sources in the hands of Mr. Ashwin.




20. Dividend received by a real estate investment trust (REIT) from special purpose vehicle (SPV) and distributed to its unit holders is –

(a) exempt in the hands of both the REIT and the unit holders unconditionally

(b) exempt in the hands of the REIT only if the SPV is a specified domestic company; exempt in the hands of unit holders only if taxable in the hands of REIT

(c) exempt in the hands of the REIT only if the SPV is a specified domestic company; exempt in the hands of unit holders only if exempt in the hands of REIT

(d) exempt in the hands of the REIT only if the SPV is a specified domestic company; exempt unconditionally in the hands of unit holders




21. Which of the following can be treated as "profits derived from" business or undertaking to qualify for deduction under section 80‐IB? (i) Transport subsidy (ii) Duty drawback receipts (iii) interest subsidy (iv) power subsidy (a) Only (ii) (b) (ii) & (iii) (c) (i), (iii) & (iv) (d) All the above




22. ABC Ltd., an Indian company engaged in manufacture of steel, has incurred expenditure on advertisement in a souvenir of a political party. Which of the following statements are correct? (a) Such expenditure is allowable as deduction while computing its business income. (b) Such expenditure is not allowable as deduction while computing its total income. (c) Such expenditure is not allowable as deduction while computing its business income but is allowable as deduction from gross total income. (d) Such expenditure is neither allowable as deduction from business income nor allowable as deduction from gross total income




23. Rental income earned from the business of letting out of properties is – (a) always taxable as income from house property (b) always taxable as business income (c) taxable as business income or income from house property, at the option of the assessee. However, the practice should be followed consistently. (d) taxable as business income only if the entire or substantial income of the assessee was from letting out of property. Otherwise, the same would be taxable as income from house property.




24. ABC Ltd., an Indian company commenced business on 1.2.2019. It incurred preliminary expenses of Rs.35 lakhs during the period from 1.4.2018 to 31.1.2019. The cost of the project is Rs.5 crore. The following are the details as on 31.3.2019: Issued Share Capital ‐ Rs.3 crore; Share Premium ‐ Rs.50 lakhs; Debentures ‐ Rs.1 crore; Long‐term borrowings ‐ Rs.2 crore. The deduction under section 35D for P.Y.2018‐19 is ‐ (a) Rs.5 lakhs (b) Rs.6 lakhs (c) Rs.6.50 lakhs (d) Rs.7 lakhs




25. Himalaya Ltd. is an eligible start‐up engaged in eligible business. Its gross total income included profits of Rs.25 lakhs from such business. The Assessing Officer made disallowance of Rs.3 lakhs under section 40(a)(ia) and of Rs.2 lakhs under section 43B. The deduction allowable under section 80‐IAC would be – (a) Rs.25 lakhs (b) Rs.28 lakhs (c) Rs.30 lakhs (d) Rs.20 lakhs




26. Delta Limited is engaged in growing and manufacturing rubber in India. It commenced its operations from 1st April, 2018. It acquired plant and machinery (second hand), factory building and furniture at a cost of Rs.62 lakhs, Rs.37 lakhs and Rs.8 lakhs, respectively, in the P.Y. 2018‐19 by way of ECS through bank account. Assuming that all the assets were put to use for more than 180 days during the P.Y. 2018‐19, you are required to compute the written down value of each block as on 1st April, 2019. (a) Rs.52.70 lakhs; Rs.33.30 lakhs & Rs.7.20 lakhs (b) Rs.58.75 lakhs; Rs.35.71 lakhs & Rs.7.72 lakhs (c) Rs.58.28 lakhs;Rs.35.52 lakhs & Rs.7.68 lakhs (d) Rs.59.675 lakhs; Rs.36.075 lakhs & Rs.7.8 lakhs






27. An Assessing Officer entered a hotel run by a person, in respect of whom he exercises jurisdiction, at 8.30 p.m. for the purpose of collecting information, which may be useful for the purposes of the Act. The hotel is kept open for business every day between 8 a.m. and 10 p.m. As per the provisions of section 133B, (a) The A.O. cannot enter the premises at 8.30 p.m. since it is after sunset (b) The A.O. can enter premises at 8.30 p.m. and take away books of account kept at the hotel after taking prior approval of the Principal Chief Commissioner or Chief Commissioner. (c) The A.O. can enter premises at 8.30 p.m. and take away books of account kept at the hotel after recording reasons for doing so. (d) The A.O. can enter premises at 8.30 p.m. but cannot take away books of account kept at the hotel






28. In the course of search operations under section 132 in the month of May, 2019, Mr. Aakash makes a declaration under section 132(4) on the earning of income not disclosed in respect of P.Y. 2018‐ 19. He also explains the manner in which he has derived such income and he pays the tax together with interest on such income and declares such income in the return of income filed by him in the month of July, 2019. Is penalty leviable in this case? (a) No penalty is attracted since Mr. Aakash has voluntarily made a declaration under section 132(4). (b) Penalty@10% of undisclosed income would be attracted even if Mr. Aakash has voluntarily made a declaration under section 132(4). (c) Penalty@30% of undisclosed income would be attracted even if Mr. Aakash has voluntarily made a declaration under section 132(4).


(d) Penalty@60% of undisclosed income would be attracted even if Mr. Aakash has voluntarily made a declaration under section 132(4).






29. ABC Ltd. took on sub‐lease a building from Ms. Jhanvi with effect from 1.7.2018 on a rent of Rs.20,000 per month. It also took on hire machinery from Ms. Jhanvi with effect from 1.10.2018 on hire charges of Rs.15,000 per month. ABC Ltd. entered into two separate agreements with Ms. Jhanvi for sub‐lease of building and hiring of machinery. Which of the following statements are correct with reference to ABC Ltd.'s liability to deduct tax at source, assuming that one‐month's rent was received as security deposit, which is refundable at the end of the lease period? (a) No tax needs to be deducted at source since rent for building does not exceed Rs.1,80,000 p.a. and rent for machinery also does not exceed Rs.1,80,000 p.a. Security deposit refundable at the end of the lease term is not rent for the purpose of TDS (b) Tax has to be deducted@10% on rent of Rs.2,00,000 (including security deposit) for building, but no tax needs to be deducted on rent for machinery (including security deposit), since the same does not exceed Rs.1,80,000. (c) Tax has to be deducted@10% on Rs.2,00,000 and @2% on Rs.1,05,000 (i.e. rent including security deposit) (d) Tax has to be deducted@10% on Rs.1,80,000 and @2% on Rs.90,000. Security deposit refundable at the end of the lease period is not rent.






30. Mudra Adco Ltd., an advertising company, has retained a sum of Rs.15 lakhs, towards charges for procuring and canvassing advertisements, from payment of Rs.1 crore due to Cloud TV, a television channel, and remitted the balance amount of Rs.85 lakhs to the television channel. Which of the following statements are correct? (a) No TDS is attracted on the sum of Rs.15 lakhs retained by Mudra Adco Ltd. (b) TDS@2% is attracted on the sum of Rs.15 lakhs retained by Mudra Adco Ltd (c) TDS@5% is attracted on the sum of Rs.15 lakhs retained by Mudra Adco Ltd. (d) TDS@10% is attracted on the sum of Rs.15 lakhs retained by Mudra Adco Ltd.






31. Mr. X acquired a house property at Mumbai from Mr. Y, a resident, for a consideration of Rs.90 lakhs, on 20.6.2018. On the same day, Mr. X made two separate transactions, thereby acquiring an urban plot in Kolkata from Mr. C for a sum of Rs.49,50,000 and rural agricultural land from Mr. D for a consideration of Rs.60 lakhs. Which of the following statements are correct? (a) No tax deduction at source is required in respect of any of the three payments. (b) TDS@1% is attracted on all the three payments. (c) TDS@1% on Rs.90 lakhs and Rs.49,50,000 are attracted. No TDS on payment of Rs.60,00,000 for acquisition of rural agricultural land (d) TDS@1% on Rs.90 lakhs is attracted. No TDS on payments of Rs.49,50,000 and 60,00,000 32. A notified infrastructure debt fund eligible for exemption under section 10(47) of the Income‐tax Act, 1961 pays interest of Rs.5 lakhs to a company incorporated in a foreign country. The foreign company incurred expenditure of Rs.12,000 for earning such interest. The fund also pays interest of Rs.3 lakhs to Mr. Frank, who is a resident of Country A, a notified jurisdictional area. Which of the following statements are correct? (a) No tax deduction at source is required in respect of both the payments. (b) No TDS is required in respect of payment of Rs.5 lakhs to the foreign company. However payment of interest to Frank attracts TDS@31.2% (c) TDS@5.20% is attracted on Rs.4,88,000 to the foreign company. TDS@31.2% is attracted on interest payment of Rs.3 lakhs to Mr. Frank (d) TDS@5.20% is attracted on interest payment of Rs.5 lakhs to the foreign company. TDS@31.2% is attracted on interest payment of Rs.3 lakhs to Mr. Frank




33. Mr. Ram, a resident individual aged 55 years, has not furnished his return of income for A.Y.2018‐19. However, the total income assessed in respect of such year under section 144 is Rs.12 lakh. Determine the quantum of penalty leviable under section 270A? (a) Penalty leviable is Rs.1,79,400, being tax payable on total income of Rs.12 lakh (b) Penalty leviable is Rs.89,700, being 50% of tax payable on Rs.12 lakh (c) Penalty leviable under section 270A is Rs.53,300, being 50% of tax payable on under‐reported income of Rs.9.50 lakhs (i.e., Rs.12 lakhs ‐ basic exemption limit of Rs.2.50 lakhs) (d) Penalty leviable under section 270A is Rs.1,06,600, being tax payable on under‐reported income of Rs.9.50 lakhs (i.e., Rs.12 lakhs ‐ basic exemption limit of Rs.2.50 lakhs)




34. A private bank has not filed its statement of financial transaction or reportable account in relation to the specified financial transactions for the financial year 2018‐19. A notice was issued by the prescribed income‐tax authority on 1st October, 2019 requiring the bank to furnish the statement by 31st October, 2019. The bank, however, furnished the statement only on 15th November, 2019. What would be the penalty leviable under section 271FA? (a) Rs.91,500 (b) Rs.13,600 (c) Rs.16,800 (d) Rs.22,800 35. Raman & Associates had made payment of Rs.2 lakh to the contractors for carrying out labour job work at various sites, but had not deducted tax at source. What are the tax consequences? (a) Rs.2,00,000 has to be added back while computing business income. However, no penalty is leviable (b) Rs.60,000 has to be added back while computing business income. However, no penalty is leviable (c) Penalty equal to the amount of tax which has not been deducted is leviable in this case. Also, Rs.2,00,000 has to be added back while computing business income. (d) Penalty equal to the amount of tax which has not been deducted is leviable in this case. Also, Rs.60,000 has to be added back while computing business income




36. Mr. Aryan is constructing a residential house property in Mumbai for self‐occupation. He has taken a loan of Rs.35 lakhs on 30.3.2018 for this purpose. He pays interest of Rs.3 lakhs during the P.Y.2018‐19. He repays Rs.3 lakhs towards principal on 31.3.2019. The construction is completed in April, 2019. This is the only house property of Mr. Aryan. For A.Y.2019‐20, (a) Mr. Aryan is entitled for deduction of Rs.2 lakhs under section 24 and Rs.1.50 lakhs under section 80C (b) Mr. Aryan is entitled for deduction of Rs.2 lakhs under section 24, Rs.50,000 under section 80EE and Rs.1.50 lakhs under section 80C. (c) Mr. Aryan is neither entitled for deduction under section 24 nor under section 80C. He is, however, entitled for deduction of Rs.50,000 under section 80EE. (d) Mr. Aryan is not entitled for deduction under section 24, section 80C and section 80EE. 37. PQR Ltd., a domestic company, has distributed on 15/10/2018, dividend of Rs.230 lakh to its shareholders. On 17/9/2018, PQR Ltd. has received dividend of Rs.60 lakh from its domestic subsidiary company XYZ Ltd., on which XYZ Ltd. has paid dividend distribution tax under section 115‐ O. The additional income‐tax payable by PQR Ltd. under section 115‐O is – (a) Rs.29.70 lakhs (b) Rs.34.944 lakhs (c) Rs.34.608 lakhs (d) Rs.29.42 lakhs 38. Lima Ltd., a domestic company, purchases its own unlisted shares on 13th August, 2018. The consideration for buyback amounted to Rs.23 lakh, which was paid on the same day. The amount received by the company two years back for issue of such shares determined in the manner specified in Rule 40BB was Rs.17 lakh. The additional income‐tax payable by Lima Ltd. is – (a) Rs.1,03,824 (b) Rs.1,04,832 (c) Rs.1,39,776 (d) Rs.1,38,432 39. A REIT derives rental income of Rs.2 crore from real estate property directly owned by it and short term capital gains of Rs.1 crore on sale of developmental properties. It also receives interest income of Rs.3 crore from Gamma Ltd., an Indian company, in which it holds controlling interest. The REIT holds 80% of the shareholding of Gamma Ltd. Which of the following statements is correct? (a) All the above income are taxable in the hands of REIT (b) REIT enjoys pass through status in respect of the above income and hence, such income are taxable in the hands of the unit holders. (c) REIT enjoys pass through status in respect of interest income from Gamma Ltd. and hence, such income is taxable in the hands of the unit holders. Rental income and short‐term capital gains are taxable in the hands of the REIT (d) REIT enjoys pass through status in respect of interest income from Gamma Ltd. and rental income from directly owned real estate property and hence, such income are taxable in the hands of the unit holders. Short‐term capital gains is taxable in the hands of the REIT




40. Mr. Hari has income of Rs.52 lakhs under the head “Profits and gains of business or profession”. One of his businesses is eligible for deduction@100% of profits under section 80‐IB for A.Y. 2019‐20. The profit from such business included in the business income is Rs.20 lakhs. The tax payable by Mr. Hari (rounded off), assuming that he has no other income during the P.Y.2018‐19, is – (a) Rs.8,03,400 (b) Rs.10,89,950 (c) Rs.9,90,860 (d) Rs.11,00,530




41. The assessment of Satpura Ltd. was completed under section 143(3) with an addition of Rs.18 lakhs to the returned income. Satpura Ltd. preferred appeal before the Commissioner (Appeals) which is pending now. Which of the following statements is incorrect? (a) The A.O. can initiate reassessment proceedings in respect of income chargeable to tax which has escaped assessment, provided such income which has escaped assessment does not form part of the additions of Rs.18 lakhs to the returned income, which is the subject matter of appeal. (b) The A.O. can pass an order under 154(1) to rectify a mistake apparent from the record, provided the rectification is in relation to a matter, other than the matter which has been considered and decided in the appeal before Commissioner (Appeals). (c) Under section 264, the Commissioner can revise the order pending before the Commissioner (Appeals), if the revision pertains to a matter, other than the matter(s) covered in the appeal before Commissioner (Appeals) (d) Under section 263, if the order is prejudicial to the interests of the revenue, the Commissioner can revise the order pending before the Commissioner (Appeals), if the revision pertains to a matter, other than the matter(s) covered in the appeal before Commissioner (Appeals) 42. Kamala charitable trust, registered u/s 12AA, having its main object as medical relief, earned dividend income of Rs.3 lakhs, income of Rs.2 lakhs from mutual funds registered under section 10(23D) and agricultural income of Rs.4 lakhs during the P.Y.2018‐19. Which of the following statements is correct? (a) The trust has to apply such income for charitable purposes as per the provisions of section 11 to claim exemption in respect of such income. (b) The trust can claim exemption under section 10(1), 10(34) and 10(35) in respect of its agricultural income, dividend and income from mutual funds, respectively, without applying such income for charitable purposes. (c) The trust can claim exemption under sections 10(34) and 10(35) in respect of its dividend and income from mutual funds, respectively, without applying such income for charitable purposes. However, it cannot claim exemption under section 10(1) in respect of agricultural income without applying such income for charitable purposes. (d) The trust can claim exemption under section 10(1) in respect of its agricultural income without applying such income for charitable purposes. However, it cannot claim exemption in respect of its income from mutual funds registered under section 10(23D) and dividend income of Rs.3 lakhs without applying such income for charitable purposes.




43. Delta Ltd., a domestic company, declared dividend of Rs.85 lakh for the year F.Y.2018‐19 and distributed the same on 27.6.2019. Mr. Ganesh, holding 15% shares in Delta Ltd., receives dividend of Rs.12.75 lakh in June, 2019. Mr. Rajesh, holding 10% shares in Delta Ltd., receives dividend of Rs.8.50 lakh. Which of the following statements is correct?

(a) Dividend distribution tax u/s 115‐O is attracted in the hands of Delta Ltd. There would be no tax on dividend received by Mr. Ganesh and Mr. Rajesh in their individual hands.

(b) Dividend distribution tax u/s 115‐O is attracted in the hands of Delta Ltd. However, dividend received by Mr. Ganesh and Mr. Rajesh is also taxable in their individual hands

(c) Dividend distribution tax is payable by Delta Ltd. u/s 115‐O. Also, dividend received by Mr. Ganesh is taxable in his hands@10%.

(d) Dividend distribution tax is payable by Delta Ltd. u/s 115‐O. Also, Mr. Ganesh has to pay tax@10% on dividend of Rs.2.75 lakhs received by him.




44. Which of the following individuals would be entitled to opt for presumptive taxation schemes under the Income‐tax Act, 1961 for A.Y.2019‐20?

(i) A retail trader having turnover of Rs.2 crore during the previous year 2018‐19

(ii) A practising CA having gross receipts of Rs.92 lakhs during the previous year 2018‐19.

(iii) A wholesale trader having turnover of Rs.1.96 crore during the previous year 2018‐19.

(iv) A doctor having gross receipts of Rs.50 lakhs during the previous year 2018‐19

(v) Individual owning 8 goods carriages as on 1.4.2018. He sold 2 goods carriages on 1.5.2018 and purchased 4 goods carriages on 1.7.2018.




(a) Only (iii) (b) (iii) & (v) (c) (i), (iii), (iv) & (v) (d) (i), (ii), (iii), (iv) & (v)




45. Mr. Arjun’s, aged 40 years, total income comprises of long‐term capital gains on sale of land Rs.5 lakhs; short‐term capital gains on sale of STT paid listed equity shares Rs.2 lakhs; income from lottery Rs.1 lakh and savings bank interest Rs.30,000. He invests Rs.1.50 lakhs in PPF. His tax liability for A.Y.2019‐20 is – (a) Rs.1,64,800 (b) Rs.1,66,400 (c) Rs.1,14,400 (d) Rs.1,13,300 46. Mrs. Kavitha, wife of Mr. Sundar, is a partner in a firm. Her capital contribution of Rs.5 lakhs to the firm as on 1.4.2018 included Rs.3 lakhs contributed out of gift received from Sundar. On 2.4.2018, she further invested Rs.1 lakh out of gift received from Sundar. The firm paid interest on capital of Rs.60,000 and share of profit of Rs.50,000 during the F.Y.2018‐19. The entire interest has been allowed as deduction in the hands of the firm. Which of the following statements is correct? (a) Share of profit is exempt but interest on capital is taxable in the hands of Mrs. Kavitha (b) Share of profit is exempt but interest of Rs.40,000 is includible in the income of Mr. Sundar and interest of Rs.20,000 is includible in the income of Mrs. Kavitha (c) Share of profit is exempt but interest of Rs.36,000 is includible in the income of Mr. Sundar and interest of Rs.24,000 is includible in the income of Mrs. Kavitha (d) Share of profit to the extent of Rs.30,000 and interest on capital to the extent of Rs.36,000 is includible in the hands of Mr. Sundar 47. X Ltd., a domestic company, has a total income of Rs.10,01,00,000 for A.Y.2019‐20. The gross receipts of X Ltd. for P.Y.2016‐17 is Rs.240 crore. The tax liability of X Ltd. for A.Y.2019‐20 is (a) Rs.2,76,55,500 (b) Rs.2,79,24,000 (c) Rs.3,46,42,610 (d) Rs.3,49,78,940




48. M/s. Atlanta Airlines, incorporated as a company in USA, operated its flights to India and vice versa during the year 2018‐19 and collected charges of Rs.280 crores for carriage of passengers and cargo, out of which Rs.100 crores were received in US Dollars for the passenger fare from Atlanta to Delhi. Out of Rs.100 crores, US dollars equivalent to Rs.40 crores is received in India. The total expenses for the year on operation of such flights were Rs.11 crores. The effective rate of income‐tax applicable on total income of M/s. Atlanta Airlines is

(a) 42.432%

(b) 43.68%

(c) 43.26%

(d) 42.024%




49. Abhinav, an individual aged 52 years resident in India, bought 3,000 equity shares of Rs.10 each of Theta Ltd. at Rs.70 per share on 1.6.2018. He sold 1,800 equity shares at Rs.50 per share on 3.11.2018 and the remaining 1,200 shares at Rs.60 per share on 23.3.2019. Theta Ltd. declared a dividend of 40%, the record date being 14.8.2018. On 15.3.2019, Abhinav sold a house from which he derived a long‐term capital gain of Rs.1,25,000. Assuming Abhinav’s interest income from bank fixed deposit is Rs.3,00,000, his tax liability (rounded off) for A.Y.2018‐19 would be (a) Rs.18,620 (b) Rs.19,920 (c) Rs.20,110 (d) Rs.18,440






50. Ms Geetha, born in USA, comes to India for the first time on 5.1.2019 and left India on 28.5.2019. She was born and bought up in the USA but her grandparents were born in Karachi before the year 1940. In © The Institute of Chartered Accountants of India December, 2015, she bought, in foreign currency, 500 Global Depository Receipts of PQR Ltd, an Indian Company, which were issued in accordance with the notified scheme of the Central Government against the initial issue of shares of PQR Ltd. In January, 2018, she sold 300 GDRs outside India to Mr. Frank, a citizen and resident of Country ‘X’ and 200 GDRs to Mr. Kamal, a Resident but not ordinarily resident in India. Comment on the tax consequences of such sale transaction under the Income-tax Act, 1961 – (a) Capital gains arising on sale of 500 GDRs shall be subject to tax @20% with indexation benefit in India (b) No capital gains would arise on sale of 500 GDRs in India, since the GDRs are purchased in foreign currency (c) No capital gains would arise on sale of 300 GDRs, but capital gains arising on sale of 200 GDRs shall be taxed in India @10% without indexation benefit (d) No capital gains would arise on sale of 300 GDRs, but capital gains arising on sale of 200 GDRs shall be taxed @20% with indexation benefit in India



Section - D Enterprise Information System

Total question are six.

Question 1 and 6 are compulsory questions and Question 1 is multiple choice question having 1.5 marks each.






There will be no negative marking for wrong answers. There will be no internal or external choice in them however the candidate is required to answer three questions from remaining four questions.










Question 1






(a) ___________ is an umbrella term that includes the applications, infrastructure and tools, and best practices that enable access to and analysis of information to improve and optimize decisions and performance.






(i) Business Intelligence


(ii) Operating system


(iii) Software


(iv) Google










(b) _______ is an attempt to gain access to other's network without authorization.






(i) Cyber crime


(ii) Cyber war


(iii) Hacking


(iv) None of the above






(c) ________ is a link between hardware and software.






(i) Wire


(ii) Linkage


(iii) Operating system


(iv) Front end






(d) Which of the following is the best defines of risk?






(i) Undesired events are prevented


(ii) Inherent vulnerability is identified


(iii) Physical threats are documented


(iv) Threats exploits vulnerability






(e) Which of the following is not a core banking service






(i) Advances


(ii) Letter of credit


(iii) Reporting


(iv) Deposits






(f) Which of the following is not an attribute of information






(i) Availibility


(ii) Mode and format


(iii) Completeness


(iv) Inadequacy






(g) xbrl is used by






(i) Investors


(ii) Government


(iii) All of the above


(iv) None of the above






(h) ERP stands for?






(i) Enterprise resource planning


(ii) Enterprise rating point


(iii) Envoromental retail programme


(iv) Enviroment report presentation






(i) Which of these is not an example of Relation data base






(i) Acces


(ii) Java


(iii) MySQL


(iv) Oracle






(j) Which of these is not a mobile OS






(i) Android


(ii) IOS


(iii) Symbian


(iv) None of the above






2. (a) What are modules of an ideal ERP? 6 marks


(b) What is difference types of Networking systems. 4 Marks






3.(a) List out internal controls for implementation of CBS in branch. 6 Marks


(b) List out various control objectives of e-commerce or m-commerce. 4 Marks






4. (a) Draw a flowchart based on following information. 6 Marks






If purchase mode is online then 2% discount


If purchase mode is via app then 5% discount


If direct purchase from store then no discount but loyalty point shall be given for purchase more than 500 Rs.






(b) What can be the various risks involved with use of credit card. 4 Marks










5. (a) Draw a workflow diagram of m-commerce and describe various steps and corresponding activities involved in this diagram. 6 Marks


(b) Explain front end and back end of a software 4 Marks






OR






Explain coding errors. 4 Marks






6. What are the benefits of Automating business processes. 5 Marks.

Comments

Post a Comment

Popular posts from this blog

Suggested answers November 2018 - Advance accounting Old Course IPCC

November 2018 Following are answers, a care is taken to answer them correctly, however, if any mistakes are identified by you, kindly share with me, I would love to hear them and will incorporate changes accordingly. Question Covered in post 1(a), 1(b), 1(c), 1(d), 2, 3(a),  3(b), 4, 5(a), 5(b),  6(b), 7(a), 7(b), 7(c),  7(d), 7(e) Question Pending 6(a) 1(a) (i) Annual lease rent = Rs. 32,500 (ii) Income = 26,000, 32,500, 39,000 (iii) Depreciation = Rs. 20,000, Rs. 25,000, Rs. 30,000 1(b) (i) Prior period Item adjustment Prior Period A/c Dr. To Salary Payable Salary payable A/c Dr To Cash Prior period item shall be disclosed separately (ii) Wages with retrospective effect It is not taken as error or omission in the preparation of Financial statements and hence this is not a prior period item, additional liability of Rs. 75,000 shall be included in current year Salary 1(c) (i) Present obligation as a result of a past obligating ev

Similarities between Delhi and Sikkim

 Delhi and Sikkim are although very different in terms of culture, tradition and food however both have some similarities. The first similarity is the area, both Delhi and Sikkim are small where as Sikkim ranks twenty seventh in terms of covered area and similarly Delhi ranks thirty first in terms of covered area. There is high literacy rate in Delhi and Sikkim, Delhi literacy rate is 86.21% and Sikkim's literacy rate is 82.6% which is also thirteenth in rank. Delhi and Sikkim have high human development index, Delhi ranks Fifth with 0.746 HDI and Sikkim ranks tenth with 0.716 HDI. Both Delhi and Sikkim comes in Northern portion of India and shares almost same latitude. Both Delhi and Sikkim are great heritage of culture and language hub.